NURS 221 TEST 2

¡Supera tus tareas y exámenes ahora con Quizwiz!

A nurse is counseling a middle adult who describes having difficulty dealing with several issues. Which of the following problems the client verbalized should the nurse identify as the priority for further assessment and intervention? A. "I am struggling to accept that my parents are aging and need so much help" B. "It's been so stressful for me to think about having intimate relationships" C. "I now I should volunteer my time for a good cause, but maybe I'm just selfish" D. "I love my grandchildren, but my son expects me to relive my parenting days"

"It's been so stressful for me to think about having intimate relationships" ; when using the urgent vs. non-urgent approach to client care, the nurse determines that the counseling priority is the problem that reflects a lack of completion of the previous stage and progression to the current stage of development. According to Erikson, it is a task of the young adulthood to develop intimacy vs. isolation. This middle adults is still struggling with this task and needs assistance in working through searching for and developing intimate relationships with others.

Potassium

-(K+) major cation in ICF -Plays a vital role in cell metabolism; transmission of nerve impulses; functioning of cardiac, lung, and muscle tissues; and acid-base balance -Potassium has a reciprocal action with Sodium -EXPECTED SERUM POTASSIUM LEVELS ARE **3.5-5 mEq/L

Sodium

-(Na+) major electrolyte found in ECF and is present in most body fluids or secretions -Essential for maintenance of acid-base and fluid balance, active and passive transport mechanisms, and irritability and conduction of nerve and muscle tisssue -EXPECTED SERUM SODIUM LEVELS ARE BETWEEN **136-145 mEq/L

Treatment for Hypokalemia

-Administer IV potassium, but never IV bolus, no more than 10-20 mEq/hr -Dilute K+ to a concentration of no more than 1mEq K+ to 10 mL solution and infuse slowly, no faster than 10 mEq/hr -Monitor for phlebitis (tissue irritant). Discontinue the IV, and notify the provider for infiltration of K+

Key concerns for adolescents

-Continue with scoliosis screening -At risk for: -eating disorders -mood disorders -and self-harm, -STIs -unplanned pregnancy -Prone to sports injuries and injury from other activities such as trampoline use -Begin driving- education regarding vehicle safety, drivers' ed., no texting while driving

Risk Factors for Hyponatremia

-Deficient ECF volume -Excessive GI loss: vomiting, NG suctioning, diarrhea, tap water enemas -Renal Losses: diuretics, kidney disease, adrenal insufficiency, excessive sweating -Skin losses: burns, wound drainage, GI obstruction, peripheral edema, ascites -Increased or normal ECF volume: excessive oral water intake, syndrome of inappropriate antidiuretic hormone (SIADH) -Edematous states: HF, cirrhosis, nephrotic syndrome -Excessive hypotonic fluids -Inadequate sodium intake (NPO status) -Hyperglycemia

Risk Factors for Hyperkalemic pts

-ECF Shift: insufficient insulin, acidosis (diabetic ketoacidosis) tissue catabolism (sepsis, trauma, surgery, fever, MI) -Kidney failure, severe dehydration, potassium-sparing diuretics, ACE inhibitors, and adrenal insufficiency

Key concerns for young adults

-Especially at risk for: -STIs -unplanned pregnancy -periodontal disease -violent death -work-related injury -Monitor nutrition and physical activity -monitor calcium intake for women

Treatment for Hypernatremia

-Fluid loss- tx with hypotonic fluids to rehydrate cells (0.225% sodium chloride) -If caused by excess sodium, administer loop diuretic to increase renal sodium excretion and discourage sodium intake

Treatment for Hyponatremia

-Fluid overload=treat with fluid restriction -Acute hyponatremia= treat with hypertonic fluids and add Na to diet -Administer hypertonic IV therapy (3% sodium -Loss of relatively more salt than water, cells swell by osmosis, risk for cerebral edema, hypertonic fluids to shrink cells and dilute ECF

Calcium

-Found in the body's cells, bones, and teeth. -Essential for proper functioning of the cardiovascular, neuromuscular, and endocrine systems, as well as blood clotting and bone and teeth formation -EXPECTED LEVELS TO BE **9-10.5 mg/dL

Risk factor assessments

-Genetics: predisposition to diseases -Gender -Physiologic states: BMI, pregnancy -Environmental and life-style factors -Age

Risk Factors for Hypokalemic pts

-Hyperaldosteronism -Prolonged administration of non-electrolyte-containing IV solutions such as 5% Dextrose in water -Excessive GI losses: vomitting, NG suctioning, diarrhea, excessive laxative use -Renal Losses: excessive use of K+ excreting diuretics, such as furosemide, corticosteroids -Skin Losses: diaphoresis, wound losses -ICF: metabolic alkalosis, after correction of acidosis (treatment of diabetic ketoacidosis), during tissue repair (burns, trauma, starvation), total parenteral nutrition

Patient education points

-Make sure patient is ready (grieving process) -Environment -Psychomotor (hands-on) skills are best taught with hands-on practice -There is an affective (feelings) part of learning- support groups -Ask client for preferences for cognitive learning -Education must be documented

Findings/ S&S for Hypocalcemia

-Numbness and tingling (fingers and around mouth -Hyperactive Deep Tendon Reflexes -Positive Chvostek's sign (tapping on the facial nerve triggering facial twitching) -Positive Trousseau's sign (hand/finger spasms with sustained BP cuff inflation) -Frequent, painful muscle spasms at rest that can progress to tetany -Weak, thready pulse, tachy or bradycardia -Prolonged QT interval and ST segments -Seizures due to overstimulation of the CNS

Treatment for Hypocalcemia

-Oral or IV calcium supplements (carefully monitor respiratory and cardio status) -initiate seizure precautions -Encourage foods high in calcium, including dairy products and dark green veggies

Key concerns for toddlers

-Picky eaters who need 24-30 oz. of milk per day -Prefer finger foods -Cut food into small pieces -Same safety concerns as infants, but they are more mobile -Start to teach safety, such as being wary of strangers, not touching hot objects

Findings/ S&S for Hypokalemic pts

-Premature Ventricular Contractions, bradys, blocks, Ventricular Tachycardia, ST depression, flattening Twaves -Weak irregular pulse, HTN and Ortho HTN, and respiratory distress -Hypoactive bowel sounds, constipation, nausea, vomiting, and anorexia -Anxiety which can progress to lethargy

Key concerns for school age children

-Scoliosis screening is recommended -This group is responsible for their own dental health, and sometimes dietary choices -Substance abuse education

Hyperkalemia

-Serum K+ level greater than 5.0 mEq/L -Increased intake of K+, movement of K+ out of the cells, or inadequate renal excretion, -Uncommon in pts with adequate kidney function -Life-threatening due to risk of cardiac arrhythmias and cardiac arrest

Hypernatremia

-Serum Na level greater than 145 mEq/L -Increased Na levels cause hypertonicity of the serum. Cells become dehydrated, bc the water shifts out of the cells

Hypokalemia

-Serum potassium level less than 3.5 -Result of an increased loss of potassium from the body, decreased intake and absorption of potassium, or movement of potassium into the cells

Hyponatremia

-Serum sodium levels below 136 mEq/L -Results from an excess of water in plasma OR loss of sodium-rich fluids -Delays and slows the depolarization of membranes -Water moves from ECF -> (into) ICF, causes cells to swell (cerebral edema) -Serious complications include coma, seizures, and respiratory arrest.

Findings/ S&S for Kyperkalemic pts

-V-fib, peaked T, wide QRS, cardiac arrest, diarrhea, cramps -Slow irregular pulse; hypotension

Expected Findings/Signs &Symptoms for Hyponatremia pts

-VS: hypothermia, tachycardia, rapid thready pulse, hypotension, and orhtostatic hypotension -Neuromusculoskeletal: headache, confusion, lethargy, muscle weakness with possible respiratory compromise, fatigue, decreased deep tendon reflexes, seizures, and comas -GI: Increased motility, hyperactive bowel sounds, abdominal cramping, anorexia, nausea, and vomiting

Key concerns for preschoolers

-Vision screenings are standard before starting school -Need a forward facing car seat until at least age 4, booster seats after -Safety: wear helmets, swimming safety

Risk Factors for Hypernatremia pts

-Water deprivation -Heat stroke -Excessive Na intake: dietary sodium intake, hypertonic IV fluids, hypertonic tube feedings, bicarbonate intake -Excessive Na retention: kidney failure, Cushing's, aldosteronism, and some glucocorticosteroids -Fluid Losses: fever, diaphoresis, burns, respiratory infection, diebetes, hyperglycemia, watery diarrhea -Compensatory Mechanism: increased thirst and increased production of ADH

Key concerns for infants

-aspiration -burns -bodily harm -drowning -falls -poisoning -MVA -suffocation

Treatment for Hyperkalemia

-fluids with dextrose and reg. insulin- insulin takes glucose into the cell, K goes with it (promotes the movement of K+ from ECF to ICF) -To increase potassium excretion: administer loop diuretics, such as furosemide, which increase the excretion of potassium from the renal system

Six rights of medication administration

-right client -right medication -right dose -right time -right route -right documentation

Expected Findings/Signs &Symptoms for Hypernatremia

-tachycardia, orthostatic HTN, thirst, dry mucous membranes, warm flushed skin, oliguria, and restlesness

How often should a women get a routine physical exam?

1-3 years and BMI at every visit

A parent brings a 5 month old infant to the clinic for well-infant check. The infant weighed 3.2 kg (7 lb) at birth. If the infant has followed the usual pattern of growth for 5 months, how much should the infant weigh? (round up to the nearest tenth)

14.5 lb: the infant should gain 0.7 kg (1.5 lb) per month in the first 6 months. 1.5 lb x age (5 months) + birthweight (7 lb) = 14.5 lb

What is the highest % of caloric intake that should come from fats?

30%

With each gram of a carbohydrate, how many kcal's does it produce?

4 kcal's

With each gram of a protein, how many kcal's does it produce?

4 kcal's

A patient has finished a 16 oz container of orange juice. The intake and output sheet documents fluid in milliliters. What should the nurse document as intake?

480 mL; 1 fluid oz = 30 mL

An acronym for early signs of hypoxia is? ATRIA

A - anxiety T - tachycardia R - restlessness I - irritable A - apprehension

What is complete protein?

A complete protein, also called a high-quality protein, contains all essential amino acids in sufficient quantity to support growth and maintain nitrogen balance.

What is an example of preload?

A hemorrhage would cause a decrease

Hemothorax

A hemothorax is an accumulation of blood and fluid in the pleural cavity between the parietal and visceral pleurae, usually owing to a trauma.

A nasal cannula

A nasal cannula is a simple, comfortable device used for precise oxygen delivery. Nasal cannulas and oxygen masks deliver oxygen therapy, which improves the levels of tissue oxygenation.

Expiration

A passive process in respiration

Pneumothorax

A pneumothorax is a collection of air in the pleural space.

What are diet considerations for older adults? (over 65 years)

A slower metabolic rate requires fewer calories Thirst sensations diminish Older adults need the same amount of most vitamins and minerals as younger adults Calcium may be necessary and is important for both men and women Many older adults require carbohydrates that provide fiber and bulk to enhance gastrointestinal funtion

A nurse is teaching a client how to administer medication through a jejunostomy tube. Which of the following instructions should the nurse include in the teaching? A. "Flush the tube before and after each medication" B. Administer your medication with your enteral feeding" C. "Administer tablets through the tube slowly" D. "Mix all the crushed medications prior to dissolving in water"

A. "Flush the tube before and after each medication" ; The client should flush the tubing before and after each medication with 15-30 mL of water to prevent clogging of the tube

A nurse is cautioning the mother of an 8 month old infant about safety. Which of the following statements by the mother indicates an understanding of safety for the infant? A. "My baby loved to play with his crib gym, but I took it away from him." B. "I just bought a soft mattress so my baby will sleep better" C. "My baby really likes sleeping on the fluffy pillow we just got for him" D. "I just bought a child-safety gate that folds like an accordion"

A. "My baby loved to play with his crib gym, but I took it away from him." Parents should remove gyms and mobiles by 4 months because injury can occur from choking or strangulations

A nurse is talking with parents of a school age child who describe several issues that concern them. Which of the following problems the parents verbalized should the nurse identify as the priority for further assessment and intervention? A. "We just don't understand why our son can't keep up with the other kids in simple activities like running and jumping" B. "Our son keeps trying to find ways around our household rules. He always wants to make deals with us" C. "We think our son is trying to hard to excel in math just to get the top grades in his class" D. "Our son is always afraid the kids in school will laugh at hime because he likes to sing and write little poems"

A. "We just don't understand why our son can't keep up with the other kids in simple activities like running and jumping" ; When using the urgent vs. non-urgent approach to client care, the priority issues is the problem that reflects a lack of completion of the previous stage of development and progression to the current stage of development. According to Erikson, it is a task of the preschool stage to develop initiative vs. guilt. This school age child is still trying to develop the physical abilities he needs to feel a sense of accomplishment. He is still struggling with this task and needs assistance with motor skills and agility.

A nurse is preparing to administer a medication to client. The medication was scheduled for administration at 0900. Which of the following are acceptable administration times for this medication? (select all that apply) A. 0905 B. 0825 C. 1000 D. 0840 E. 0935

A. 0905; a medication should be administered within 30 min of the scheduled time. D. 0840; a medication should be administered within 30 min of the scheduled time.

A nurse on a medical-surgical unit is caring for a group of clients. The nurse should identify which of the following clients is at risk for hypovolemia? A. A client who has NG suctioning B. A client who has chronic constipation C. A client who has syndrome of inappropriate antidiuretic hormone D. A client who took an overdose of sodium bicarbonate antacids

A. A client who has NG suctioning The nurse should identify that a client who has NG suctioning is at risk for hypovolemia due to excessive GI losses

A nurse is preparing to perform endotracheal suctioning for a client. What is the appropriate guidelines for the nurse to follow? (Select all that apply) A. Apply suction while withdrawing the catheter B. Perform suctioning on a routine basis, every 2-3 hours C. Maintain medical asepsis during suctioning D. Use a new catheter for each suctioning attempt E. Limit suctioning to two to three attempts

A. Apply suction while withdrawing the catheter; the nurse should apply suction pressure only while withdrawing the catheter, not while inserting it D. Use a new catheter for each suctioning attempt; The nurse should not reuse the suction catheter unless an inline suctioning system is in place E. Limit suctioning to two to three attempts; to prevent hypoxemia, the nurse should limit each suctioning session to two to three attempts and allow at least 1 minute between passes for ventilation and oxygenation

A nurse is caring for a client who has a tracheostomy. What actions should the nurse take each time he provides tracheostomy care? (Select all that apply) A. Apply the oxygen source loosely if the SPO2 decreases during the procedure B. Use surgical asepsis to remove and clean the inner cannula C. Clean the outer surfaces in a circular motion from the stoma site outward D. Replace the tracheostomy ties with new ties E. Cut a slit in gauze square to place beneath the tube holder

A. Apply the oxygen source loosely if the SPO2 decreases during the procedure; the nurse must be prepared to provide supplemental oxygen in response to any decline in oxygen saturation while performing tracheostomy care B. Use surgical asepsis to remove and clean the inner cannula; the nurse should use a sterile disposable tracheostomy cleaning kit or sterile supplies and maintain surgical asepsis throughout this part of the procedure C. Clean the outer surfaces in a circular motion from the stoma site outward; this helps move mucus and contaminated material away from the stoma for easy removal

A nurse is planning diversionary activities for children on an inpatient pediatric unit. Which of the following should the nurse incorporate as appropriate play activities for preschoolers? (select all that apply) A. Assembling puzzles B. Pulling wheeled toys C. Using musical toys D. Using finger paints E. Coloring with crayons

A. Assembling puzzles; putting puzzles together is appropriate for preschoolers and helps develop fine motor and cognitive skills C. Using musical toys; playing with musical toys is appropriate for preschoolers and helps develop fine motor skills and coordination E. Coloring with crayons; using crayons to color on paper or in a coloring book is appropriate for preschoolers and helps develop fine motor skills and coordination

A nurse is planning diversionary activities for children on an inpatient pediatric unit. Which of the following should the nurse incorporate as appropriate play activities for school-age children? (select all that apply) A. Building models B. Playing video games C. Reading books D. Using toy carpentry tools E. Shaping modeling clay

A. Building models; building simple models is appropriate for school age children and helps develop fine motor and cognitive skills B. Playing video games; playing video games, especially educational and nonviolent ones, is appropriate for school age children and helps develop fine and cognitive skills. C. Reading books; reading books is appropriate for school age children and helps develop cognitive and communication skills.

A nurse at an elementary school is planning a health promotion and primary prevention class. Which of the following topics are appropriate to include for the parents of school age children? (select all that apply) A. Childhood obesity B. Substance use disorders C Scoliosis screening D. Front seat seatbelt use E. Stranger awareness

A. Childhood obesity; parents of school age children need to be aware of nutritional strategies for preventing childhood obesity B. Substance use disorders; parents of school age children need to know how to teach children to say no to illegal drugs, alcohol, and all other harmful or addictive substances C. Scoliosis screening; school age children and adolescents require screening for scoliosis E. Stranger awareness; parents need to reinforce stranger safety as soon as their children are old enough to understand it, and throughout all stages of childhood

A nurse is talking with the parents of a toddler. Which of the following should the nurse suggest regarding discipline? A. Establish consistent boundaries B. Place him in a room with the door closed C. Have him learn by trial and error D. Use favorite snacks as rewards

A. Establish consistent boundaries; toddlers need to have consistent boundaries for discipline to be effective

A nurse is a clinic is caring for a client who has multiple risk factors for cardiovascular disease. When planning health promotion and disease prevention strategies for this client, which of the following interventions should the nurse include? (Select all that apply) A. Help the client see the benefits of her actions B. Identify the client's support systems C. Suggest and recommend community resources D. Devise and set goals for the client E. Teach stress management strategies

A. Help the client see the benefits of her actions; the nurse should help the client recognize the benefits of her health-promoting actions while also overcoming barriers to taking implementing actions B. Identify the client's support systems; once the nurse has collected information about who can help the client change her unhealthful behaviors, she can suggest ways the client's supportive friends and family can get involved C. Suggest and recommend community resources; the nurse should promote the client's use of any available community or online resources that can help her progress toward meeting her goals E. Teach stress management strategies; stress is a contributing factor to cardiovascular disease, as well as many other specific and systemic disorders

A nurse educator is teaching a module on safe medication administration to newly hire nurse. Which of the following statement by a newly hired nurse indicates understanding of the nurse's responsibility when implementing medication therapy? (select all that apply) A. I will observe for medication side effects B. I will monitor for therapeutic effects C. I will prescribe the appropriate dose D. I will change the dose if adverse effect occur E. I will refuse to give a medication if I believe it is unsafe

A. I will observe for medication side effects; the nurse is responsible for observing for medication side effects. This is within a nurse's scope of practice B. I will monitor for therapeutic effects; the nurse is responsible for monitoring for therapeutic effects. This is within the nurse's scope of practice. E. I will refuse to give a medication if I believe it is unsafe; the nurse is responsible for recognizing if a medication could potentially harm a client. It is within the nurse's scope of practice to refuse to administer the medication and contact the provider.

A nurse is caring for a 5-year-old client whose parents report that she fears painful procedures, such as injections. Which of the following strategies should the nurse use to try to help ease the child's fear? (select all that apply) A. Invite the child to assist with mealtime activities B. Cluster invasive procedures whenever possible C. Assign caregivers with whom the child is familiar D. Have the parents bring in a favorite toy from home E. Engage the child in pretend play with a toy medical kit.

A. Invite the child to assist with mealtime activities; preschoolers enjoy mastering tasks they can perform independently. Assisting with routine, nonthreatening tasks can help improve their self-esteem during hospitalization D. Have the parents bring in a favorite toy from home; having familiar and cherished objects nearby is therapeutic for children during their hospitalization E. Engage the child in pretend play with a toy medical kit; pretend play helps children determine the difference between reality and fantasy (imagine fears), especially with the assistance of the nurse during hospitalization

A nurse is giving a presentation about accident preventions to a group of parents of toddlers. Which of the following accident-preventions strategies should the nurse include? (select all that apply) A. Keep toxin agents locked in cabinets B. Keep toilet seat up C. Turn pot handles toward the back of the stove D. Place safety gates across stairways E. Make sure balloons are fully inflated

A. Keep toxin agents locked in cabinets; parents must prevent toddlers from accessing dangerous substances C. Turn pot handles toward the back of the stove; if toddlers can reach a pot handle, they can pull the pot and its contents down on themselves and incur serious injuries D. Place safety gates across stairways; at the bottom of a staircase, they prevent toddlers from climbing and falling backward. At the top of a staircase, they prevent toddlers from falling down the stairs

A nurse is collecting history and physical examination data from a middle adult. The nurse should expect to find decreases in which of the following physiologic functions? (select all that apply) A. Metabolism B. Ability to hear low-pitched sounds C. Gastric secretion D. Far vision E. Glomerular filtration

A. Metabolism; it declines and weight gain is likely C. Gastric secretion; decreases in secretions of bicarbonate and gastric mucus begin and persist into older age. This increased the risk of peptic ulcer disease E. Glomerular filtration; decline in the rates

A nurse in a senior center is counseling a group of older adults about their nutritional needs and considerations. What information should the nurse include? (Select all that apply) A. Older adults are more prone to dehydration than younger adults B. Older adults need the same amount of most vitamins and minerals as younger adults do C. Many older men and women need calcium supplements D. Older adults need more calories than they did when they were younger E. Older adults should consume a diet low in carbohydrates

A. Older adults are more prone to dehydration than younger adults are, because sensations of thirst diminish with age, leaving older adults more prone to dehydration. B. Older adults need the same amount of most vitamins and minerals as younger adults do, because these requirements do not change from middle adulthood to older adulthood. C. Many older men and women need calcium supplementation, because they may ingest insufficient calcium in the diet and many need supplements to help prevent bone demineralization (osteoporosis).

A nurse is receiving a provider's prescription by telephone for a client who is reporting moderate to severe pain. Which of the following nursing actions are appropriate? (Select all that apply) A. Repeat the details of the prescription back to the provider B. Have another nurse listen to the telephone prescription C. Obtain the prescriber's signature on the prescription within 24 hours D. Decline the verbal prescription because it is not an emergency situation E. Tell the charge nurse that the provider has prescribed morphine by telephone

A. Repeat the details of the prescription back to the provider; the nurse should repeated the medication's name, dosage, time or interval, route, and any other pertinent information back to the provider and receive and document confirmation B. Have another nurse listen to the telephone prescription; having another nurse listen to the telephone prescription is a safety precaution that helps prevent medication errors due to miscommunication C. Obtain the prescriber's signature on the prescription within 24 hours; the provider must sign the prescription within the time frame the facility specifies in its policies (generally 24 hours)

A nurse is assessing a client who has an acute respiratory infection that puts her at risk for hypoxemia. What are early indications that should alert the nurse that the client is developing hypoxemia? (Select all that apply) A. Restlessness B. Tachypnea C. Bradycardia D. Confusion E. Pallor

A. Restlessness B. tachypnea D. confusion E. pallor along with tachycardia, elevated blood pressure, use of accessory muscles, nasal flaring, tracheal tugging, and adventitious lung sounds

A nurse is talking with the parents of a 6-month old infant about gross motor development. Which of the following gross motor skills are expected finding in the next 3 months? (select all that apply) A. Rolls from back to front B. Bears weight on legs C. Walks holding onto furniture D. Sits unsupported E. Sits down from a standing position

A. Rolls from back to front; the infant should be able to roll from back to front by 6 months B. Bears weight on legs; the infant should be able to bear weight on legs by 7 months D. Sits unsupported; the infant should be able to do this by 8 months

A nurse is preparing a wellness presentation for families at a community center. When discussing health screenings for adolescents, which of the following information about scoliosis should the nurse include? A. Scoliosis is more common among girls than it is among boys B. Loss of height is often the first sign of scoliosis C. Scoliosis screening is essential during the adolescent growth spurt D. Slouching is a common cause of scoliosis, especially in adolescents E. Scoliosis is a forward curvature of the spine

A. Scoliosis is more common among girls than it is among boys; girls are more likely than boys to have adolescent idiopathic scoliosis C. Scoliosis screening is essential during the adolescent growth spurt; idiopathic scoliosis is most noticeable during the adolescent growth spurt

A nurse in a health clinic is caring for a 21-year-old client who reports a sore throat. The client tells the nurse that he has not seen a doctor since high school. Which of the following health screenings should the nurse expect the provider to perform for this client? A. Testicular examination B. Blood glucose C. Fecal occult blood D. Prostate-specific-antigen

A. Testicular examination; starting at age 20, examinations for testicular cancer are appropriate, along with blood pressure, and body mass index measurements and cholesterol determinations

The skin barrier covering a client's intestinal fistula keeps falling off when she stands up to ambulate. The nurse has reapplied it twice during the current shift, but it remains intact only when the client is supine in bed. The nurse telephoned the physical therapist about the difficulties containing the drainage from the fistula, so the therapist did not ambulate the client today. The client sat in a chair during lunch with an absorbent pad over the fistula. The client ate all the food on her tray. The wound care nurse confirmed that she will see the client later today. The client states she feels frustrate at not having physical therapy, but the nurse thinks the client welcomed having a day to rest. Which of the following information should the nurse include in the change-of-shift report? (Select all that apply) A. The physical therapist did not ambulate the client today B. The skin barrier's seal stays on in bed but loosens when the client stands C. The client seemed to welcome having a "day off" from physical therapy D. The wound care nurse will see the client later today E. The client ate all the food on her lunch tray

A. The physical therapist did not ambulate the client today; the oncoming nurse needs to know about any changes in or deviations from the clients plan of care, such as missing a physical therapy session B. The skin barrier's seal stays on in bed but loosens when the client stands; the current problem about the adhesion of the skin barrier is important information the oncoming nurse needs to know and address D. The wound care nurse will see the client later today; the oncoming nurse needs to know about any consultations that will take place during the shift

A mother tells the nurse that her 2-year-old child has temper tantrums. The client says "no" every time the mother tries to help her get dressed. The nurse explains that, developmentally, the toddler is A. Trying to increase her independence B. Developing a sense of trust C. Manifesting an anger management problem D. Attempting to finish a project she started

A. Trying to increase her independence; toddlers express a drive for independence by opposing the desires of those in authority and attempting to do everything themselves

A nurse is reviewing the medical record of a client who has hypocalcemia. The nurse should identify which of the the following findings as risk factors for the development of this electrolyte imbalance. A. crohn's disease B. postoperative following appendectomy C. history of bone cancer D. hyperthyroidism

A. crohn's disease is arisk factor for hypocalcemia. this malabsorption disorder places the client at risk for hypocalcemia due to inadequate calcium absorption

A nurse is reviewing the CDC for immunizations for young adults. Which of the following recommendations should the nurse include in this discussion? (select all that apply) A. human papillomavirus B. Measles, mumps, rubella C. Varicella D. Haemophilus influenzae type b E. Polio

A. human papillomavirus B. Measles, mumps, rubella C. Varicella

A nurse is planning care for a client who has hypernatremia. Which of the following actions should the nurse anticipate including in the plan of care? A. infuse hypotonic IV fluids B. implement a fluid restriction C. increase sodium intake D. administer sodium polystyrene sulfonate

A. infuse hypotonic IV fluids Hypotonic IV fluids, such as 0.225% sodium chloride, are indicated for the tx of hypernatremia related to fluid loss to expand the ECF volume and rehydrate the cells

A nurse prepares an injection of morphine (Duramorph) to administer to a client who reports pain. Prior to administering the medication, the nurse is called to another room to assist another client onto a bedpan. She asks the second nurse to give the injection. Which of the following actions should the second nurse take? A. offer to assist the client needing the bedpan B. administer the injection prepared by the other nurse C. prepare another syringe and administer the injection D. tell the client needing the bedpan she will have to wait for her nurse

A. offer to assist the client needing the bedpan; the second nurse should offer to assist the client needing the bedpan. This will allow the nurse who prepared the injection to administer it

Compliance

Ability of the lungs to distend or expand in response to increased intra-alveolar pressure

What processes does absorption use?

Absorption uses the processes of active transport, passive diffusion, osmosis, and pinocytosis.

Tx for Hypomagnesemia

Administer IV or PO magnesium sulfate (monitor closely for diarrhea), encourage food high in magnesium (Include whole grains and dark green veggies, seizure precautions

Tx for FVD

Administer isotonic fluids (isotonic solutions such as lactated Ringer's or 0.9% sodium chloride; blood transfusions), po fluids as tolerated

How does age affect nutrition and metabolism?

Age - affects nutritional requirements

What is airway resistance?

Airway resistance is the increase in pressure that occurs as the diameter of the airways decreases from mouth/nose to alveoli. Any further decrease in airway diameter by bronchoconstriction can increase airway resistance.

Can an albumin level be used as a diagnostic marker for nutritional status?

Albumin level is a poor short-term indicator of protein status, because albumin is not sensitive to acute changes in nutritional status. It's a long half life (21 days) makes it a better indicator of chronic illness states than a current protein status at any given point in time

What is altered cardiac output?

Altered cardiac output: Insufficient volume is ejected into the systemic and pulmonary circulation; the result of left-sided or right-sided heart failure

Maintenance and promotion of lung expansion

Ambulation and positioning reduces pulmonary stasis, maintains ventilation and oxygenation Incentive spirometry encourages voluntary deep breathing Noninvasive ventilation maintains positive airway pressure and improves alveolar ventilation

What is stroke volume?

Amount of blood ejected from the left ventricle with each contraction

Inspiration

An active process stimulated by chemical receptors in the aorta

What is the simplest form of a protein?

An amino acid

When is an artificial airway used?

An artificial airway is used for a patient with a decreased level of consciousness or an airway obstruction and aids in removal of tracheobronchial secretions.

What is Anorexia nervosa?

An eating disorder characterized by: Body weight less than 85% of ideal Fear of being fat Feeling fat With female clients, no menses for at least 3 consecutive months

What is Bulimia nervosa?

An eating disorder characterized by: a cycle of binge eating followed by purging (vomiting, using diuretics, or laxatives, exercise, fasting) lack of control during binges average at least two binges per week for at least 3 months

What does an electrocardiogram (ECG) reflect and monitor?

An electrocardiogram (ECG) reflects the electrical activity of the conduction system. An ECG monitors the regularity and path of the electrical impulse through the conduction system; however, it does not reflect the muscular work of the heart. The normal sequence on the ECG is called the normal sinus rhythm (NSR). NSR implies that the impulse originates at the SA node and follows the normal sequence through the conduction system.

What is an ideal body weight? (IBW)

An ideal body weight (IBW) provides an estimate of what a person should weigh.

An oxygenation mask

An oxygen mask is a plastic device that fits snugly over the mouth and nose and is secured in place with a strap.

What is anabolism?

Anabolism is the building of more complex biochemical substances through synthesis of nutrients.

What is anthropometry?

Anthropometry is a measurement system of the size and makeup of the body.

How does appetite affect nutrition and metabolism?

Appetite - decreases with illness, medications, pain, depression, and unpleasant environment stimuli

Systemic circulation

Arteries and veins deliver nutrients and oxygen and remove waste products.

Why do artificial airways need to stay in the correct position?

Artificial airways need to stay in the correct position to prevent airway damage.

Key concerns for older adults

At risk for: -cardiovascular disease -arthritis -osteoporosis -falls -dementia -diabetes -cancer -pain -social isolation -Continue with DEXA screening for osteoporosis -eye exam for glaucoma -mental health for depression Nutrition: -impaired taste -lack of resources -prescribed diets which are unappetizing -Injury prevention related to sensory impairment and impaired mobility

Key concerns for middle adults

At risk for: -obesity -cardiac disease -cancer -substance abuse -stress Start DEXA screening for osteoporosis -eye exam for glaucoma -mental health for depression -Vitamin D and calcium supplementation

What follows the P wave?

Atrial contraction

Physical examination for oxygenation: Ausculate

Auscultation: Normal and abnormal heart and lung sounds listen to all lung sounds - it is more effective and thorough

A nurse at a provider's office is talking with a 45 years old client who has no specific family history of cancer or diabetes mellitus about planning her routine screenings. Which of the following statements indicates that the client understands how to proceed? A. "So I don't need the colon cancer procedure for another 2 - 3 years." B. "For now, I should continue to have a mammogram each year." C. "Because the doctor just did a Pap smear, I'll come back next year for another one." D. "I had my blood glucose test last year, so I won't need it again till next year."

B. "For now, I should continue to have a mammogram each year." Between the ages of 40 and 50 women should have a mammogram annually.

A nurse is teaching an adult client how to administer ear drops. Which of the following statements by the client indicates understanding of the proper technique? A. "I will straighten my ear canal by pulling my ear down and back" B. "I will gently apply pressure with my finger to the tragus of my ear after putting in the drops" C. "I will insert the nozzle of the ear drop bottle snug into my ear before squeezing the drops in" D. After the drops are in, I will place a cotton ball all the way into my ear canal"

B. "I will gently apply pressure with my finger to the tragus of my ear after putting in the drops" ; the client should gently apply pressure with the finger to the tragus of the ear after administering the drops to help the drops go into the ear canal

The mother of a 7-month-old infant tells the nurse at the pediatric clinic that her baby has been fussy with occasional loose stools since she started feeding him fruits and vegetables. Which of the following responses by the nurse are appropriate? (select all that apply) A. "It might be good to add bananas, as they can help with loose stools" B. "Let's make a list of the foods he is eating so we can spot any problem" C. "Did the changes begin after you started one particular food?" D. "Has he been vomiting since he started these new foods?" E. "Most babies react with a little indigestion when you start new foods"

B. "Let's make a list of the foods he is eating so we can spot any problem"; before the nurse can determine that there is a problem, such as a food allergy or intolerance, she should determine the components of the child diet C. "Did the changes begin after you started one particular food?"; fussiness and diarrhea, as well as a rash and vomiting or constipation, can all be signs of a food allergy or intolerance. Before the nurse can intervene, she has to collect data that can help her plan the appropriate interventions D. "Has he been vomiting since he started these new foods?"; vomiting and constipation can also be signs of a food allergy or intolerance. Before the nurse can intervene, she has to college data that can help her plan the appropriate interventions

A nurse is talking with parents of a preschooler who describe several issues that concern them. Which of the following problems the parents verbalized should the nurse identify as the priority for further assessment and intervention? A. "Our son will only eat a few things, like burgers and bananas, and pretty much refuses everything else" B. "Our son has these temper tantrums every time we tell him to do something he doesn't want to do" C. "We think our son truly believes that his toys have personalities and talk to him, especially at night" D. "We feel bad when we see our son trying so hard to button his shirt. We just tell him this is something he'll just have to learn"

B. "Our son has these temper tantrums every time we tell him to do something he doesn't want to do" ; When using the urgent vs non-urgent approach to client care, the nurse determines that the priority issues is the problem that reflects a lack of completion of the previous stage of development and progression to the current stage of development. According to Erikson, it is a task of the toddler stage to develop autonomy vs. shame and doubt. This preschooler is still acting out with negativism, which is a persistent negative response to requests, often manifested in tantrums. He is still struggling with this task and needs assistance in working through that stage.

A nurse educator is teaching a module on pharmacokinetics to a group of newly licensed nurses. Which of the following statements by a newly licensed nurse indicates an understanding of the first-pass effefct? A. "Some medications block normal receptor activity regulated by endogenous compounds or receptor activity caused by other medications" B. "Some medications may have to be administered by a non-enteral route to avoid inactivation as they travel through the liver" C. Some medications leave the body more slowly and therefore have a greater risk for medication accumulation and toxicity" D. Some medications have a wide safety margin, so there is no need for routine serum medication level monitoring"

B. "Some medications may have to be administered by a non-enteral route to avoid inactivation as they travel through the liver" ; some medications are inactivated on their first pass through the liver and must be given by a non-enteral route to prevent this inactivation. These medications are usually given by routes such as sublingual or IV

A nurse ins a med-surg unit is caring for a group of clients, which of the following clients should the nurse anticipate a prescription for fluid restriction? A. a client who has a new diagnosis of adrenal insufficiency B. a client who has HF C. a client who is receiving tx for diabetic ketoacidosis D. a client who has abdominal ascites

B. A client who has HF The nurse should anticipate a client who has HF to require fluid and sodium restriction to reduce the workload on the heart

A nurse in a provider's office is collecting data from the mother of a 1 year old child. The client states that her child is old enough for toilet training. Following an educational session by the nurse, the client now states that her earlier ideas have changed. She is now willing to postpone toilet training until the child is older. Learning has occurred in which of the following domains? A. Cognitive B. Affective C. Psychomotor D. Kinesthetic

B. Affective; Affective learning has taken place, as evidence by the client's changed ideas about toilet training

A nurse is evaluating how well a client learned the information presented in an instructional session about following a heart-healthy diet. The client states that she understands what to do now. Which of the following actions by the nurse should assist the nurse in evaluating the clients learning? A. Encourage the client to ask questions B. Ask the client to explain how to select or prepare meals C. Encourage the client to fill out an evaluation form D. Ask the client if she has resources for further instruction on this topic.

B. Ask the client to explain how to select or prepare meals; a useful strategy for evaluating learning is to ask the client to explain in her own words how she will implement what she learned

A nurse is caring for a client who is having difficulty breathing. The client is lying in bed and is already receiving oxygen therapy via nasal cannula. What intervention is the nurse's priority? A. Increase the oxygen flow B. Assist the client to Fowler's position C. Promote removal of pulmonary secretions D. Obtain a specimen for arterial blood gases

B. Assist the client to Fowler's position; the priority action the nurse should take when using the airway, breathing, circulation (ABC) approach to care delivery is to relieve the client's dyspnea (difficulty breathing). Fowler's position facilitates maximal lung expansion and thus optimizes breathing. With the client in this position, the nurse can better assess and determine the cause of the client's dyspnea.

A nurse is planning care for a client who is dehydrated, which of the following actions should the nurse include? A. administer antihypertensive on schedule B. check the clients weight every morning C. notify the provider of a urine output greater than 30 mL/hr D. encourage independent ambulation four times a day

B. Check the clients weight every morning

A nurse is collecting data from an older adult client as part of a comprehensive physical examination. Which of the following findings should the nurse expect as changes associated with aging? (select all that apply) A. Skin thickening B. Decreased height C. Increased saliva production D. Nail thickening E. Decreased bladder capacity

B. Decreased height; with aging, height decreases due to the thinning of intervertebral disks D. Nail thickening; aging brings thickening of the nails of the fingers and toes, and also changes their shape, color, and growth rate E. Decreased bladder capacity; while young adults have a bladder capacity of about 500 to 600 mL, older adults have a capacity of about 250 mL

A nurse is caring for a client who is at high risk for aspiration. What is an appropriate nursing intervention? A. Give the client thin liquids B. Instruct the client to tuck her chin when swallowing C. Have the client use a straw D. Encourage the client to lie down and rest after meals

B. Instruct the client to tuck her chin when swallowing; this allows the food to pass down the esophagus more easily.

A nurse is teaching a young adult client about health promotion and illness prevention. Which of the following statements made by the client indicates an understanding of the teaching? A. "I already had my immunizations as a child, so I'm protection in that area" B. It is important to schedule routine health care visits even if I am feeling well" C. "If I am having any discomfort, I'll just go to an urgent care center" D. "If I am feeling stressed, I will remind myself that this is something I should expect"

B. It is important to schedule routine health care visits even if I am feeling well" ; young adulthood is a time of relative health, but routine screenings and health care visits are still important

A nurse is talking with a client who recently attended a cholesterol screening event and a heart-healthy nutrition presentation at a neighborhood center. His total cholesterol result from the screening was 248 mg/dL, so he saw his provider and received a medication prescription to improve his cholesterol level. The client was later hospitalized for severe chest pain, and subsequently enrolled in a cardiac rehabilitation program. Which of the following activities of this client is an example of primary prevention? A. Cholesterol screening B. Nutrition presentation C. Medication therapy D. Cardiac rehabilitation

B. Nutrition presentation; primary presentation encompasses strategies that actually help prevent illness or injury. This level of prevention includes health information about nutrition, exercise, stress management, and protection from injuries and illness.

A nurse is planning a presentation to a group of older adults at a senior community center about the essential screening tests and preventive procedures during this stage of life. Which of the following should the nurse include? (select all that apply) A. Human papilloma virus (HPV) immunization B. Pneumococcal immunization C. Eye examination D. Mental health screening E. Dual-energy x-ray absorptionmetry (DEXA) scanning

B. Pneumococcal immunization; older adults are expecially susceptible to pneumococcal infections, so this is an essential preventive measure for this stage of life C. Eye examination; screening for glaucoma via regular eye examination is essential for older adults D. Mental health screening; screening for depression via mental health assessments is essential for older adults E. Dual-energy x-ray absorptionmetry (DEXA) scanning; screening for osteoporosis via DEXA scanning is essential for older adults

A nurse is reviewing care-seat safety with parents of a 1 month old infant. When reviewing car-seat use, which of the following instructions should the nurse include? A. Use a care seat that has three-point harness system B. Position the car seat so that the infant is rear-facing C. Secure the care seat in the front passenger seat of the vehicle D. Put soft padding in the car seat behind the infant's back and neck

B. Position the car seat so that the infant is rear-facing; infants in a car seat in the front passenger seat are at risk for injury from the airbag in the event of a crash

A nursing instructor is reviewing documentation with a group of nursing students. Which of the following legal guidelines should they follow when documenting in a client's record? (Select all that apply) A. Cover errors with correction fluid, and write in the correct information B. Put the date and time on all entries C. Document objective data, leaving out opinions D. Use as many abbreviations as possible E. Wait until the end of the shift to document

B. Put the date and time on all entries; the day and time confirm the recording of the correct sequence of events C. Document objective data, leaving out opinions; documentation must be factual, descriptive, and objective, without opinions or criticism

When a nurse is observing a client drawing up and mixing insulin injections, which of the following best demonstrates that psychomotor learning has taken place? A. The client is able to discuss the appropriate technique B. The client is able to demonstrate the appropriate technique C. The client states that he understands D. The client is able to write the steps on a piece of paper

B. The client is able to demonstrate the appropriate technique; demonstrating the appropriate technique indicates that psychomotor learning has taken place

A nurse enters a client's room and finds him sitting in his chair. He states, "I fell in the shower, but I got myself back up and into my chair." How should the nurse document this in the client's chart? A. The client fell in the shower B. The client states he fell in the shower and was able to get himself back into his chair C. The nurse should not document this information in the chart because she did not witness the fall D. The client fell in the shower but is now resting comfortably

B. The client states he fell in the shower and was able to get himself back into his chair; by writing what the client states, the information is subjective data

A nurse is reviewing the CDC for immunizations with a middle adult client. Which of the following recommendations should the nurse include in this discussion? (select all that apply) A. Haemophilus influenzae type b B. Varicella C. Herpes zoster D. Human papilloma virus E. Seasonal influenza

B. Varicella C. Herpes zoster; typically one dose at age 60 or beyond E. Seasonal influenza

A nurse is reviewing the CDC's immunization recommendations with the parents pf an adolescent. Which of the following recommendations should the nurse include in this discussion? (select all that apply) A. Rotavirus B. Varicella C. Herpes zoster D. Human papilloma virus E. Seasonal influenza

B. Varicella; chickenpox D. Human papilloma virus; genital warts E. Seasonal influenza

A nurse is reviewing the centers for disease control and prevention (CDC's) immunization recommendations with the parents of two preschoolers. Which of the following recommendations should the nurse include in this discussion? (select all that apply) A. Haemophilus influenzae type b B. Varicella C. Polio D. Hepatitis A E. Seasonal influenza

B. Varicella; the CDC recommends a polio immunization during the preschool years C. Polio; the CDC recommends a polio immunization during the preschool years E. Seasonal influenza; the CDC recommends a polio immunization during the preschool years

A nurse is working with a newly hired nurse who is administering medications to clients. Which of the following actions by the newly hired nurse indicates understanding of medication error prevention? A. taking all medications out of the unit-dose wrappers before entering the client's room B. checking with the provider when a single dose requires administration of multiple tablets C. administering a medication, then looking up the usual dosage range D. relying on another nurse to clarify of medication prescription

B. checking with the provider when a single dose requires administration of multiple tablets; if a single dose requires multiple tablets, it is possible that an error has occurred in the transcription of the medication. This action could prevent a medication error

A nurse is collecting data from a client who has hypercalcemia as a result of long-term use of glucocorticoids, which of the following findings should the nurse expect? Select all that apply A. hyperreflexia B. confusion C. Positive Chvostek's sign D. Bone Pain E. nausea and vomiting

B. confusion D. bone pain E.nausea and vomiting

A nurse on a pediatric unit is caring for an adolescent who has multiple fractures. Which of the following interventions are appropriate for this client? (select all that apply) A. suggest that his parents room in with him B. provide a television and DVD's for him to watch C. Limit visitors to immediate family D. Devise a regular schedule for inpatient routines E. Allow him to perform his own morning care

B. provide a television and DVD's for him to watch; nonviolent DVD's are appropriate diversional activities for an adolescent E. Allow him to perform his own morning care; allowing him to perform his own morning care helps promote a sense of independence

A nurse is reviewing safety precautions with a group of young adults at a community health fair. Which of the following recommendations should the nurse indicate specifically for this age group? (select all that apply) A. Install bath rails and grab bars in bathrooms B. wear a helmet while skiing C. install a carbon monoxide detector D. secure firearms in a safe location E. remove throw rugs from the home

B. wear a helmet while skiing; wearing a helmet while skiing helps reduce the risk of head injury. Although it applies to other age groups, many young adults engage in winter sports, so this is an age appropriate recommendation for this group C. install a carbon monoxide detector; having a carbon monoxide detector in the home is essential safety precaution for young adults as well as for all other ages D. secure firearms in a safe location; securing firearms in a safe location helps reduce the risk of accidental gunshot injuries. Although this applies to all groups, many young adults own firearms, so this is an age appropriate recommendation for this developmental group

A nurse is caring for a client who weighs 80 kg (176 lbs) and is 1.6 m (5 ft 3 in) tall. Calculate her body mass index (BMI) and determine whether this client is obese based on her BMI.

BMI = weight (kg) divided by height (m squared). BMI = 80 divided by 2.56 = 31.25. A BMI above 30 identifies obesity, so this client is obese.

What is basal metabolic rate? (BMR)

Basal metabolic rate—the energy needed to maintain life-sustaining activities for a specific period of time at rest

What is body mass index? (BMI)

Body mass index (BMI) measures weight corrected for height and serves as an alternative to traditional height-weight relationships.

A nurse is talking with the parents of a 10 year old child who express concern that their son is suddenly becoming secretive, for example, closing the door when he showers, dresses, and does his homework in his room. Which of the following responses by the nurse is appropriate? A. "Perhaps you should try to find out what he is doing behind those closed doors" B. "Suggest that he leave the door ajar for his own safety" C. "At this age, children tend to become more modest and value their privacy" D. "Tell him its okay to close the door when he is undressed, but he has to do his homework where you can see him"

C. "At this age, children tend to become more modest and value their privacy" ; From a developmental perspective, it is an expression that school age children develop privacy. They have their own way of doing things and spend more time alone

A nurse is talking with an adolescent who describes having difficulty dealing with several issues. Which of the following problems the client verbalized should the nurse identify as the priority for further assessment and intervention? A. "I kind of like this girl in my class. She doesn't like me back, though, not that way" B. "I like hanging out with the guys in the science club, but the jocks pick on them" C. "I just don't seem to be any good at anything. I can't play any sports at all" D. "My dad wants me to be a lawyer like him, but I don't want to learn all that stuff"

C. "I just don't seem to be any good at anything. I can't play any sports at all" ; when using the urgent vs. non-urgent approach to client care, the nurse determines that the counseling priority is the problem that reflects a lack of completion of the previous stage and progression to the current stage of development. According to Erikson, it is a task of the school age years to develop industry (such as by learning new skills and experiencing achievements in them) vs. inferiority. This adolescent is still struggling with this task and needs assistance in working through that dilemma.

A nurse is reviewing nutritional guidelines with the parents of a 2-year-old toddler. Which of the following parent's statements indicate to the nurse that they understand the feeding guidelines for this age group? A. "I should keep feeding my son whole milk until he is 3 years old" B. "Its okay for me to give my son a cup of apple juice with each meal" C. "Ill give my son about 2 tablespoons of each food at mealtimes" D. My son loves popcorn, and I know it is better for him than sweets"

C. "Ill give my son about 2 tablespoons of each food at mealtimes"; serving sizes for toddlers should be about 1 tbsp of solid food per year of age so 2 year olds should have about 2 tbsp per serving

A nurse is preparing information for change-of-shift report. What information should the nurse include in the report? A. the client's input and output for the shift B. The client's blood pressure from the previous day C. A bone scan that is scheduled for today D. The medication routine from the medication administration record

C. A bone scan that is scheduled for today; the bone scan in important because the nurse might have to modify the client's care to accomodate leaving the unit

A nurse is providing preoperative teaching for a client who is scheduled for a mastectomy the next day. Which of the following client statements indicates that the client is ready to learn? A. "I don't want my spouse to see my incision" B. "Will you be able to give me pain medicine after the surgery?" C. Can you tell me about how long the surgery will take?" D. "My roommate listens to everything I say"

C. Can you tell me about how long the surgery will take?" Asking a concrete question about the surgery indicates that the client is ready to discuss the surgery. The client's new diagnosis of cancer may cause anxiety, fear, or depression, all of which can interfere with the learning process.

A nurse is talking with the father of a 4 year old child who states that his daughter goes to bed at 8:30 pm and wakes up at 7:30 am, but she often lies in bed talking to herself or gets up a few times before falling asleep 40 min later. At her preschool, the children take a 2 hour afternoon nap. Which of the following recommendations should the nurse make to help improve the child's sleep behavior? A. Offer the child a snack of her favorite treat right before bedtime. B. Allow the child to watch an extra 30 min of TV in the evening. C. Change the child's bedtime to 9 pm. on days she napped. D. Request that the preschool staff limit her nap to 1 hr.

C. Change the child's bedtime to 9 pm. on days she napped; preschoolers start to need less sleep than they did in previous stages. Putting the child to bed 30 min later, when she might be more tired, could help her fall asleep more readily.

A provider is discharging a client with a prescription for home oxygen therapy via nasal cannula. Client and family teaching by the nurse should include what instructions? (Select all that apply) A. Apply petroleum jelly around and inside the nares B. Remove the nasal cannula during mealtimes C. Check the position of the cannula frequently D. Report any nasal stuffiness, nausea, or fatigue E. Post "no smoking" signs in a prominent location

C. Check the position of the cannula frequently; a disadvantage of this oxygen delivery device is that it dislodges easily. The client should form the habit of checking its position periodically and readjusting it as necessary D. Report any nasal stiffness, nausea, or fatigue; oxygen toxicity is a complication of oxygen therapy, usually from high concentrations or long durations. Manifestations include a nonproductive cough, substernal pain, nasal stuffiness, nausea, vomiting, fatigue, headache, sore throat, and hypoventilation. The client should report any of these promptly E. Post "no smoking" signs in a prominent location; oxygen is combustible and thus increases the risk of fire injuries. No one in the house should smoke or use any device that might generate sparks in the area where the oxygen is in use

A nurse is caring for a 19 year old client who is sexually active and has come to the college health clinic for the first time for a checkup. Which of the following interventions should the nurse perform first to determine the client's need for health promotion and disease prevention? A. Measure the client's vital signs B. Encourage HIV screening C. Determine the client's risk factors D. Instruct the client to use condoms

C. Determine the client's risk factors; the first action the nurse should take using the nursing process in assessment. The nurse should talk with the client first to determine what risk factors the client might have before initiating the appropriate health promotion and disease prevention measures

A nurse is planning diversionary activities for children on an inpatient unit. Which of the following should the nurse incorporate as appropriate play activities for a toddler? (select all that apply) A. Building simple models B. Working with clay C. Filling and emptying containers D. Playing with blocks E. Looking at books

C. Filling and emptying containers; this activity is toddler-appropriate and helps develop fine motor skills and coordination D. Playing with blocks; this activity is toddler-appropriate and helps develop fine motor skills E. Looking at books; this activity is toddler-appropriate and helps with preparing for learning to read

A nurse is talking with an older adult about improving her nutritional status. Which of the following interventions should the nurse recommend? (select all that apply) A. Increase iron intake to prevent anemia B. Decrease fluid intake to prevent urinary incontinence C. Increase calcium intake to prevent osteoporosis D. Limit sodium intake to prevent edema E. Increase fiber intake to prevent constipation

C. Increase calcium intake to prevent osteoporosis; older adults are at risk for osteoporosis. Increasing calcium intake is one way to help prevent it D. Limit sodium intake to prevent edema; older adults are at risk for edema and hypertension. Limiting sodium intake is one way to help prevent them E. Increase fiber intake to prevent constipation; older adults should increase fiber intake to prevent constipation

A nurse is caring for a client who is 1 day post-operative following a total knee arthroplasty. The client states his pain level is 10 on a scale of 0-10. After reviewing the client's medication administration record, which of the following medications should the nurse administer? A. Meperidine (Demerol) 75 mg IM B. Fentanyl 50 mcg/hr transdermal patch C. Morphine 2 mg IV D. Oxycodone 10 mg PO

C. Morphine 2 mg IV; IV morphine is the best choice because the onset is rapid, and absorption of the medication into the blood is immediate, which provides an immediate response for a child who is reporting pain at a level of 10

A nurse is reviewing the lab test results for a client who has an elevated temp. The nurse should recognize which of the following findings as a manifestation of dehydration? (Select all that apply) A. Hct 55% B. Serum osmolarity 260 mOsm/kg C. Serum sodium 150 mEq/L D. Urine specific gravity 1.035 E. Serum creatinine 0.6 mg/dL

C. Serum sodium 150 mEq/L This serum sodium level is greater than the expected reference range of 136-145 mEq/L and is an indication of dehydration due to hemoconcentration D. Urine specific gravity 1.035 The uringe specific gravity is greater than the expected reference range of 1.005-1.030. An increased urine specific gravity is an indication of dehydration

A nurse is performing an admission assessment on a client who was hypovolemic due to vomiting and diarrhea. The nurse should expect which of the following findings? (Select all that apply) A. Distended neck veins B. Hyperthermia C. Tachycardia D. Syncope E. Decreased skin turgor

C. Tachycardia D. Syncope E. Decreased skin turgor

A nurse is caring for a client who is on a low-residue diet. The nurse should expect to see what types of food on the client's meal tray? A. Cooked barley B. Pureed broccoli C. Vanilla custard D. Lentil soup

C. Vanilla custard; a low residue diet consists of foods that are low in fiber and easy to digest. Dairy products and eggs, such as custard and yogurt, are appropriate for a low-residue diet.

A nurse is collecting data to evaluate a middle adult's psychosocial development. The nurse should expect middle adults to demonstrate which of the following capabilities? (select all that apply) A. Develop an acceptance of diminished strength and increased dependence on others. B. Feel frustrated that time is too short for attempting to start another life. C. Welcome opportunities to be creative and productive. D. Commit to finding friendships and companionship E. Become involved with community issues and activities

C. Welcome opportunities to be creative and productive; psychosocially healthy middle adults accept life's opportunities for creativity and productivity and use these opportunities for achieving Erikson's stage of generativity vs. stagnation. E. Become involved with community issues and activities; psychosocially healthy middle adults achieve Erikson's stage of generativity vs. stagnation by contributing to future generation through community involvement as well as teaching and parenting

A nurse receives a lab report for a client indicating a potassium level of 5.2 mEq/L. When notifying the provider, the nurse should anticipate which of the following actions? A. starting an IV infusion of 0.9% sodium chloride B. consulting with dietitian to increase intake of potassium C. initiating continuous cardiac monitoring D. preparing the client for gastric lavage

C. a potassium of 5.2 mEq/L indicates hyperkalemia. The nurse should anticipate the initiation of continuous cardiac monitoring due to the clients risk of dysrhythmias such as ventricular fibrillation

A nurse instructor is explaining the various stages of the lifespan to a group of nursing students. The nurse should offer which of the following behaviors by a young adult as an example of appropriate psychosocial development? A. becoming actively involved in providing guidance to the next generation B. adjusting to major changes in roles and relationships due to losses C. devoting a great deal of time to establishing an occupation D. Finding oneself "sandwiched" in between and being responsible for two generations"

C. devoting a great deal of time to establishing an occupation; exploring career options and then establishing oneself in a specific occupations is a major developmental task for a young adult

A nurse is talking with the father of a 12 year old boy who is concerned that he hasn't observed any indication that his son is approaching puberty. The nurse should explain that the first sign of sexual maturation in boys is A. the appearance of downy hair on the upper lip B. hair growth in the axillae C. enlargement of the testes and the scrotum D. deepening of the voice

C. enlargement of the testes and the scrotum; the first prepubescent change in boys is an increase in the size of the testicles along with a thinning and expanding of the scrotum

Blood tests: CBC Cardiac enzymes Cardiac troponins Serum electrolytes Cholesterol

CBC - complete blood count Cholesterol - patency of vessels

What is the cardiac output formula?

CO = stroke volume (SV) x heart rate (HR)

What happens to the lungs and thorax in a COPD patient?

COPD patient- lose elastic lung and thorax recoil As a result, the patient's work of breathing increases.

What is a CPAP used for and how does it work?

CPAP treats patients with obstructive sleep apnea, those with congestive heart failure, and preterm infants with underdeveloped lungs. In obstructive sleep apnea, airways collapse, causing shallow or absent breathing. Any air moving past the obstruction results in loud snoring.

For patients with cancer and cancer treatment, what would the medical nutrition therapy include?

Cancer and cancer treatment: Cancer is when malignant cells compete with normal cells for nutrients. Treat with education on: Anorexia, nausea, vomiting, and taste distortions are common. Malnutrition associated with cancer increases morbidity and mortality. Radiation causes anorexia, stomatitis, severe diarrhea, intestinal strictures, and pain. Nutrition management Maximize fluid and nutrient intake. Individualize diet choices to patient's needs, symptoms, and situation. Encourage small, frequent meals and snacks that are easy to digest. Term: Cachexia

How is carbon dioxide transported?

Carbon dioxide transport: Diffuses into red blood cells and is hydrated into carbonic acid

Cardioprotective nutrition

Cardioprotective nutrition = Diets rich in fiber; whole grains; fresh fruits and vegetables; nuts; antioxidants; lean meats; and omega-3 fatty

What does cardiopulmonary physiology involve?

Cardiopulmonary physiology involves delivery of deoxygenated blood (blood high in carbon dioxide and low in oxygen) to the right side of the heart and then to the lungs, where it is oxygenated. Oxygenated blood (blood high in oxygen and low in carbon dioxide) then travels from the lungs to the left side of the heart and the tissues.

Cardiopulmonary resuscitation (CPR) : in which order do you put the following regarding CPR? Airway Breathing Circulation

Cardiopulmonary resuscitation (CPR) 1. Circulation 2. Airway 3. Breathing Defibrillation (automatic external defibrillator [AED])

For patients with cardiovascular disease, what would the medical nutrition therapy include?

Cardiovascular diseases: American Heart Association (AHA) dietary guidelines: Treat with education on: Balance caloric intake with exercise. Maintain a healthy body weight. Eat a diet rich in fruits, vegetables, and complex carbohydrates. Eat fish twice per week. Limit foods and beverages high in sugar and salt. Limit trans-saturated fat to less than 1%.

What is catabolism?

Catabolism is the breakdown of biochemical substances into simpler substances; it occurs during physiological states of negative nitrogen balance.

For patients with Celiac disease, which a malabsorption syndrome, what would the medical nutrition therapy include?

Celiac disease: Gluten is contained in wheat, rye, barley, and oats. Treat with Gluten-free diet

Neural regulation components

Central nervous system Cerebral cortex

What stimulated chemoreceptors?

Changes in the chemical content of O2, CO2, and H (pH) stimulate chemoreceptors located in the medulla, aortic body, and carotid body, which in turn stimulate neural regulators to adjust the rate and depth of ventilation to maintain normal arterial blood gas levels.

Surfactant

Chemical produced in the lungs to maintain the surface tension of the alveoli and keep them from collapsing

What are chemoreceptors and what do they do?

Chemoreceptors sense changes in the chemical content and stimulate neural regulators to adjust.

Chest tube

Chest tube A catheter placed through the thorax to remove air and fluids from the pleural space, to prevent air from re-entering, or to re-establish intrapleural and intrapulmonic pressures

Imaging: Chest x-ray Cardiac catheterization

Chest x-ray - noninvasive

What is the food mass in liquefied form called?

Chyme

What could clamping the chest tube result in?

Clamping the chest tube is contraindicated. "Milking" the tubing is not routine Clamping a chest tube could possibly result in a tension pneumothorax where air pressure builds in the pleural space, collapsing the lung and creating a life threatening event.

A nurse is explaining to a group of middle adults in a community center what moral and cognitive development characteristics they should expect at this stage of life. A. List moral and cognitive development expectations during middle adulthood

Cognitive development -reaction time and speed of performance slow slightly -memory is intact -crystallized intelligence remains (stored knowledge) -fluid intelligence (how to learn and process new information) declines slightly) Moral development -spiritual beliefs and religion may take on added importance -middle adults may become more secure in their convictions -middle adults often have advance moral development

A nurse is explaining to a group of parents in a community center what cognitive development characteristics they should expect of their school age children. A. List cognitive and language development expectations during young adulthood

Cognitive development -see weight and volume as unchanging -understand simple analogies -understand time (days, seasons) -classify more complex information -understand various emotions people experience -become self-motivated -solve problems -define many words and understands rules of grammar -understand that a word may have multiple meanings -have a carbon monoxide detector in the home

A nurse on a pediatric unit is reviewing with a group of nursing students the cognitive developmental milestones to expect from adolescent clients. A. List cognitive development expectations during adolescence

Cognitive development -think at an adult level -think abstractly and deal with principles -evaluate the quality of their own thinking -have a longer attention span -are highly imaginative and idealistic -make decisions through logical operations -are future-oriented -are capable of deductive reasoning -understand how actions of an individual influence others

A nurse is explaining to the parents of a 4 month old infant what milestones they can expect their infant to achieve during this first year of her life and what they can do to encourage her development. A. Cognitive development: -name the developmental stage Piaget has identified for the first two years of life - identify three essential components that comprise this stage B. -identify toys and activities the nurse should suggest that the parents provide for their infant

Cognitive development: Piaget's sensorimotor stage (first two years) -separation -object permanence -mental representation Age-appropriate activities Toys and activities -rattles -mobiles -teething toys -nesting toys -playing pat-a-cake -playing with balls -reading books

Atelectasis

Collapse of the alveoli that prevents normal exchange of oxygen and carbon dioxide

What are complementary proteins?

Complementary proteins are pairs of incomplete proteins that, when combined, supply the total amount of protein provided by complete protein sources.

Factors affecting oxygenation: conditions affecting chest wall movement

Conditions affecting chest wall movement: Pregnancy - pushes on the diaphragm obesity - pushes on the diaphragm neuromuscular disease musculoskeletal abnormalities trauma CNS alterations

What are the key points when inserting an NG tube?

Contraindicated? Assessment? Patient position: high fowlers Medical or surgical asepsis? Checking placement

Cardiopulmonary rehabilitation

Controlled physical exercise; nutrition counseling; relaxation and stress management; medications; oxygen; compliance; systemic hydration

Coronary artery circulation

Coronary arteries supply the myocardium with nutrients and remove wastes.

A nurse is counseling an older adult who describes having difficulty dealing with several issues. Which of the following problems verbalized by the client should the nurse identify as the priority? A. "I spent my whole life dreaming about retirement, and now I wish I had my job back" B. "It's been so stressful for me to have to depend on my son to help around the house" C. "I just heard my friend Al died. That's the third one in 3 months" D. "I keep forgetting which medication I have taken during the day"

D. "I keep forgetting which medication I have taken during the day" ; the greatest risk to this client is injury from overdosing or under-dosing his medications due to his loss of short-term memory. The priority issue for the nurse is to assist the client to implement safe medication strategies. The nurse should assist the client to use a pill organizer to help him remember to take his medications and to keep a list of all current medications

A nurse is teaching a client about taking multiple oral medications at home to include time-release capsules, liquid medications, enteric-coated pills, and narcotics. Which of the following statements by the client indicates an understanding of the teaching? A. "I can open the capsule with the beads in it and sprinkle them on my oatmeal" B. "If I am having difficulty swallowing, I will add the liquid medication to a batch of pudding" C. "The pills with the coating on them can be crushed D. "I will eat two crackers with the pain pills"

D. "I will eat two crackers with the pain pills" ; it is recommended to administer irritating medications with small amounts of food. This will assist with prevention of nausea and vomiting so that the medication can be retained and take effect

A nurse is reviewing nutritional guidelines with the parents of an 11 year old child. Which of the following parents' statements should indicate to the nurse that they understand the guidelines for school age children? A. "She wants to eat as much as we do, but we're afraid she'll soon be overweight" B. "She skips lunch sometimes, but we figure its okay as long as she has a healthy breakfast and dinner" C. We limit fast-food restaurant meals to three times a week now" D. "We reward her school achievements with a point system instead of a pizza or ice cream"

D. "We reward her school achievements with a point system instead of a pizza or ice cream" ; Parents should avoid rewarding children with food for good behavior or achievements. Associations children form between food and feeling good can lead to weight problems

A nurse is preparing a presentation about basic nutrients for a group of high school athletes. She should explain to the athletes that the body's primary energy source is what? A. Fat B. Protein C. Glycogen D. Carbohydrates

D. Carbohydrates are the body's primary energy source; providing energy for cells is their primary function. They provide glucose, which burns completely and efficiently without end products to excrete, and carbohydrates are a ready source of energy, and spare proteins from depletion.

A nurse is preparing an instructional session about managing stress incontinence for an older adult. Which of the following actions should the nurse take first when meeting with the client? A. Encourage the client to participate actively in learning B. Select instrumental materials appropriate for the older adult C. Identify goals the nurse and the client agree are reasonable D. Determine what the client knows about stress incontinence

D. Determine what the client knows about stress incontinence; the first action the nurse should take using the nursing process is to assess or collect data from the client. The nurse should determine how much the client knows about stress incontinence, the accuracy of this knowledge, and what the client needs to learn to manage this condition before proceeding to instructing the client

A nurse is preparing to administer digoxin (Lanoxin) to a client who states, "I don't want to take that medication. I do not want one more pill" Which of the following responses by the nurse is appropriate in this situation? A. Your physician prescribed if for you, so you really should take it B. Well, let's just get it over quickly then C. Okay, I'll just give you your other medication D. Tell me your concerns with taking this medication

D. Tell me your concerns with taking this medication; although clients have the right to refuse a medication, the nurse is correct in determining the reason for refusal by asking the client his concerns. After gathering the client's concerns, the nurse can provide information regarding the risk of refusal and provide information, for an informed decision. At this point, if the client still exercises his right to refuse a medication, the nurse should notify appropriate personnel and document the refusal and actions taken.

A nursing instructor is explaining the various stages of the lifespan to a group of nursing students. The nurse should offer which of the following behaviors by a young adult as an example of accomplishing Erikson's tasks for psychosocial development during middle adulthood? A. The client evaluates his behavior after a social interaction B. The client states he is learning to trust others C. The client wishes to find meaningful friendships D. The client expresses concerns about the next generation

D. The client expresses concerns about the next generation; the task for a middle adult is generativity vs. stagnation. Concern for the next generation is positive sign that the middle adult is meeting a task

A nurse is providing educating for a client who has severe hypomagnesemia due to alcohol use disorder. The client is to receive magnesium sulfate. Which of the following info should the nurse include in the teaching? A. "You will receive magnesium in a series of intramuscular injections" B. "You should receive a prescription for a thiazide diuretic to take with the magnesium." C. "You should eliminate whole grains from your diet until your mag levels increases" D. "You will have your deep-tendon reflexes monitored while you are receiving magnesium."

D. The nurse should instruct the client on the need to monitor deep-tendon reflexes during administration of magnesium. This assessment helps identify hypermagnesemia that can occur during IV administration of magnesium sulfate.

Why would the patient have an NG tube?

Decompression (Salem sump) Feeding (Dubhoff) Lavage (Levin, Salem sump) Compression (Sengstaken-Blakemore)

What does the delivery of oxygen depend on?

Delivery depends on the amount of oxygen entering the lungs (ventilation), blood flow to the lungs and tissues (perfusion), the rate of diffusion, and the oxygen-carrying capacity.

What is Diabetes?

Diabetes is a disease that is focused intensely on diet and exercise with requirements limiting carbohydrate intake. Monitoring carbohydrate consumption is a key strategy in achieving glycemic control.

For patients with Diabetes mellitus, what would the medical nutrition therapy include?

Diabetes mellitus: Type 1: insulin and dietary restrictions Type 2: exercise and diet therapy initially Treat with: Individualized diet Carbohydrate control and monitoring Saturated fat less than 7% Cholesterol intake less than 200 mg/dL Protein intake 15% to 20% of diet

What should you ask your patient when assessing dietary and health history?

Dietary and health history: number of meals per day fluid intake food preferences, amounts food preparation, purchasing practices, access to food history of indigestion, heartburn, gas allergies taste chewing and swallowing appetite elimination patterns medication use activity levels religious, cultural food restrictions

Where does digestion begin? and what does it cause?

Digestion begins in the mouth and ends in the small and large intestines

What does digestion cause?

Digestion causes food to break down to simplest form for absorption, mainly in the small intestines. Ingestion is the taking in of food.

How do diseases and illnesses affect nutrition and metabolism?

Disease and illness - affects the functional ability to prepare, eat, and digest food

What can increase airway resistance? What happens to the amount of oxygen being delivered to the alveoli when the resistance increases?

Diseases causing airway obstruction such as asthma and tracheal edema increase airway resistance. When airway resistance increases, the amount of oxygen delivered to the alveoli decreases.

What causes disturbances in conduction?

Disturbances in conduction: Caused by electrical impulses that do not originate from the SA node (dysrhythmias)

For patients with diverticulitis, which is a malabsorption syndrome, what would the medical nutrition therapy include?

Diverticulitis is the inflammation of diverticula Treat Low- to moderate-residue diet for infection Treat with High fiber for chronic problems

What is obesity?

Dividing weight (in kg) by height (in m squared) determines body mass index (BMI) A BMI of 25 is upper boundary of healthy weight. Adults who have a BMI above 30 are obese

What dietary modification should an adolescent engaging in sports implement?

Drink water before and after sports activities; this prevents dehydration

What is an elemental diet?

Elemental diets are formulas with nutrients in their simplest form ready for absorption.

A nurse is caring for a patient who has impaired swallowing due to a cerebrovascular accident. What intervention should the nurse use to assist the patient with feeding?

Elevate the head of the bed 45-90 degrees. The patients head should be sufficiently elevated to prevent aspiration

What is elimination?

Elimination is when chyme is moved through peristalsis and is changed into feces

What is preload?

End-diastolic volume, effected by fluid volume

Artificial airway: When would you use endotracheal airways?

Endotracheal airways: Short-term use to ventilate, relieve upper airway obstruction, protect against aspiration, clear secretions

What does enteral nutrition consist of? (EN)

Enteral nutrition (EN) provides nutrients into the GI tract. It is physiological, safe, and economical nutritional support. Nasogastric, jejunal, or gastric tubes Surgical or endoscopic placement

Environmental pollutants?

Environmental pollutants: Second hand smoke? Work? Smoker?

What are enzymes and what do they do?

Enzymes are the catalysts that speed up chemical reactions.

What are examples of foods that are sources of protein?

Examples of foods that contain complete proteins are fish, chicken, soybeans, turkey, and cheese.

Diffusion

Exchange of respiratory gases in the alveoli and capillaries

What happens to patients who do not exercise with compromised oxygenation?

Exercise: •Thus those who do not exercise have higher pulse rates, blood pressures, and cholesterol levels; lower blood flow; and lower oxygen extraction

Fluid Volume Excess (FVE)

FVE - isotonic retention of water and sodium in high proportions. AKA Hypervolemia bc of the resulting increased blood volume Overhydration - hypoosmolar fluid imbalance, the retention of more water than electrolytes too much isotonic fluid in extracellular compartment

What factors affect energy requirements?

Factors such as age, body mass, gender, fever, starvation, menstruation, illness, injury, infection, activity level, and thyroid function affect energy requirements.

What factors affect metabolism?

Factors that affect metabolism include illness, pregnancy, lactation, and activity level.

A nurse is preparing a presentation at a community center for a group of parents who are interested in learning how to prevent childhood obesity. A. List factors the nurse should consider when incorporating ways to enhance learning. B. List barriers the nurse might encounter among the attendees.

Factors that enhance learning: -perceived benefit -cognitive and physical ability -health and cultural beliefs -active participation -age -educational level-appropriate methods Barriers to learning: -fear -anxiety -depression -physical discomfort -pain -fatigue -environmental distractions -health and cultural beliefs -sensory and perceptual deficits -psychomotor deficits

True or false: Cyanosis is an early sign of hypoxia

False, cyanosis is a late sign of hypoxia

What are fats composed of?

Fats are composed of triglycerides and fatty acids.

What are fatty acids composed of?

Fatty acids are composed of chains of carbon and hydrogen atoms with an acid group at one end of the chain and a methyl group at the other.

What is a saturated fatty acid composed of?

Fatty acids can be saturated, in which case each carbon in the chain has two attached hydrogen atoms

What is an unsaturated fatty acid composed of?

Fatty acids can be unsaturated, in which case unequal numbers of hydrogen atoms are attached, and the carbon atoms attach to each other with a double bond.

What do feces contain?

Feces contains cellulose, indigestible substances, GI tract cells, digestive secretions, water, and microbes.

What is fiber?

Fiber is a polysaccharide that is the structural part of plants not broken down by human digestive enzymes.

How does a financial situation affect nutrition and metabolism?

Financial - may prevent clients from buying foods that are high in protein, vitamins, and minerals

What do flow-oriented incentive spirometers consist of? How does it work?

Flow-oriented incentive spirometers consist of one or more plastic chambers that contain freely moving colored balls. A patient inhales slowly and with an even flow to elevate the balls and keep them floating as long as possible to ensure a maximally sustained inhalation.

Tx for FVE

Fluid restriction, diuretics, daily weights, breath sounds(lung sounds can be diminished with crackles), monitor edema

How does food move through the GI tract?

Food moves through the GI tract through peristalsis, or wavelike muscular contractions.

What is food security?

Food security means that all household members have access to sufficient, safe, and nutritious food to maintain a healthy lifestyle; sufficient food is available on a consistent basis; and the household has resources to obtain appropriate food for a nutritious diet.

What must happen to the pressure in the intrapleural space for air to flow into the lungs?

For air to flow into the lungs, intrapleural pressure becomes more negative, setting up a pressure gradient between the atmosphere and the alveoli.

What are the four cardiac chambers?

Four cardiac chambers have been identified: two atria and two ventricles.

What is a fruitarian?

Fruitarian (consumes fruit, nuts, honey, and olive oil)

How does gas move in and out of the lungs?

Gases move into and out of the lungs through pressure changes.

What gastrointestinal diseases may require medical nutrition therapy?

Gastrointestinal diseases Peptic ulcer etiology Helicobacter pylori Stress Acid overproduction Medical nutrition therapy is extremely significant in gastrointestinal diseases such as peptic ulcer, inflammatory bowel disease, and malabsorption syndrome

What does hemoglobin carry?

Hemoglobin carries O2 and CO2

The oxygen transport system

Hemoglobin, which is a carrier for oxygen and carbon dioxide, transports most oxygen (approximately 97%). The hemoglobin molecule combines with oxygen to form oxyhemoglobin. The formation of oxyhemoglobin is easily reversible, allowing hemoglobin and oxygen to dissociate (deoxyhemoglobin), which frees oxygen to enter tissues. Decreased hemoglobin levels alter the patient's ability to transport oxygen. Carbon dioxide, a product of cellular metabolism, diffuses into red blood cells and is rapidly hydrated into carbonic acid (H2CO3). The carbonic acid then dissociates into hydrogen (H) and bicarbonate (HCO3−) ions. Hemoglobin buffers the hydrogen ion, and HCO3− diffuses into the plasma. Reduced hemoglobin (deoxyhemoglobin) combines with carbon dioxide, and venous blood transports most of the carbon dioxide back to the lungs to be exhaled.

For patients with human immunodeficiency virus/acquired immunodeficiency syndrome, what would the medical nutrition therapy include?

Human immunodeficiency virus/acquired immunodeficiency syndrome: Treat with education on: Body wasting and severe weight loss and Severe diarrhea, GI malabsorption, altered nutrient metabolism are common with HIV and AIDS patients Plan: Maximize kilocalories and nutrients. Encourage small, frequent, nutrient-dense meals with fluid in between. Good hygiene and food/water safety This disease state results in several consequences that alter nutritional attainment. So... Diagnose and address each cause of nutritional depletion in the care plan. Create individually tailored nutrition support progresses in stages from oral, to enteral, and finally to parenteral. Good hand hygiene and food safety are essential because of a patient's reduced resistance to infection.

Why is water an important nutrient?

Humans are water-based systems! In all, 60% to 70% of total body weight is water. Water is critical because cell function depends on a fluid environment. Water replaces fluids lost through perspiration, elimination, and respiration All cell function depends on a fluid environment

What is an example of afterload?

Hypertension increases afterload

Hyperventilation

Hyperventilation: Ventilation in excess of that required to eliminate carbon dioxide produced by cellular metabolism

Hypoventilation

Hypoventilation: Alveolar ventilation inadequate to meet the body's oxygen demand or to eliminate sufficient carbon dioxide

Hypoxia

Hypoxia: Inadequate tissue oxygenation at the cellular level

Hypoxia: Cyanosis (central and peripheral)

Hypoxia: Cyanosis Central - see more around the face and mouth and soft palette (more consistent with hypoxia) Peripheral - related to blood flow and vasoconstriction (not evidence of hypoxia, but response to vasoconstriction)

What intervention should a nurse use at mealtime for a patient who has visual deficits?

Identify the food location as though the plate were a clock; telling the patient, for example, the chicken is at 9 o'clock and the broccoli is at 12 o'clock helps orient the patient to the items on the plate and thus facilitates independence when eating

What can happen if you suction the patient to much?

If you suction the patient too much, he or she can be at risk for hypoxemia, hypotension, dysrhythmias, and trauma to the mucosa of the lungs.

What is impaired valvular function?

Impaired valvular function: Acquired or congenital disorder of a cardiac valve by stenosis or regurgitation

What are incomplete proteins?

Incomplete proteins are missing one or more of the nine indispensable amino acids.

Risk Factors for Hypomagnesemia

Increased Magnesium Output: -GI losses (Diarrhea, NG suction) -Thiazide or loop diuretics -often associated with hypocalcemia Inadequate Magnesium intake or absorption: -Malnutrition -Alcohol use disorder -Laxative use

How does increased thickness affect diffusion across the membranes?

Increased thickness of the membrane impedes diffusion because gases take longer to transfer across the membrane. Patients with pulmonary edema, pulmonary infiltrates, or pulmonary effusion have a thickened membrane, resulting in slow diffusion, slow exchange of respiratory gases, and decreased delivery of oxygen to tissues. Chronic diseases (e.g., emphysema), acute diseases (e.g., pneumothorax), and surgical processes (e.g., lobectomy) often alter the amount of alveolar capillary membrane surface area.

Medial nutrition therapy for patients with inflammatory bowel disease: Crohn's and Idiopathic ulcerative colitis

Inflammatory bowel disease: Crohn's and Idiopathic ulcerative colitis Elemental diets for acute phase Fiber increase, Fat reduction Large meal avoidance Lactose and sorbitol avoidance Patient may need supplements or parenteral nutrition

A nurse is making safety recommendations to the parents of two preschoolers. A. List the four key areas of safety and age appropriate instructions for addressing each area.

Injury Prevention -Bodily harm ---keep firearms in a locked cabinet or container ---teach stranger safety ---wear helmets when riding a bicycle or tricycle and during any other activities that increase head-injury risk ---wear protective equipment (helmet and pads) during physical activity -Burns ---reduce the temperature setting on the hot water heater ---have smoke detectors in the home and replace the batteries regularly ---use sunscreen while outdoors -Drowning ---do not leave children unattended in the bathtub ---closely supervise children at a pool or any other body of water ---teach children to swim -Motor vehicle injuries ---use a forward facing car seat with a harness in the back seat ---if weight or height exceeds the forward facing limit, use a belt-positioning booster seat -Positioning ---avoid exposure to lead paint ---keep plants out of reach ---place safety locks on cabinets with cleaners and other chemicals ---keep a poison control number handy or program it into the phone ---keep medications in childproof containers out of reach ---have a carbon monoxide detector in the home

A nurse is reviewing safety precautions for older adults with a group of home health care nursing assistant. a. List safety recommendations for older adults

Injury prevention -install bath rails, grab bars, and handrails on stairways -teach clients about safe medication use -remove throw rugs -eliminate clutter from walkways and hallways -remove extension and phone cords from walkways and hallways -instruct about how to use ambulation-assistive devices (walkers, canes) -ensure adequate lighting -remind clients to wear eyeglasses and hearing aids -avoid drugs, including alcohol, to prevent substance use disorders -avoid driving a vehicle during or after drinking alcohol or taking drugs that impair sensory and motor functions -wear a seat belt when operating a vehicle -wear a helmet while bike riding, skiing, and other recreational activities that increase head-injury risk -install smoke and carbon monoxide detectors in the home -secure firearms in a safe location

Physical examination for oxygenation: Inspection

Inspection: Skin and mucous membranes, level of consciousness (LOC), breathing patterns, chest wall movement

Is intrapleural pressure negative or positive?

Intrapleural pressure is negative, or less than atmospheric pressure, which is 760 mm Hg at sea level.

When administering parenteral nutrition to a patient, how is is provided?

Intravenously Must be in a large vessel Placement of this catheter - initial placement (initial risk is puncturing the lungs) Major risk of infection

Fluid Volume Deficit (FVD)

Isotonic Fluid Volume Deficit - loss of water and electrolytes from the ECF. AKA Hypovolemia bc intravascular fluid is also lost Dehydration - loss of water from the body without the loss of electrolytes

What is medical nutrition therapy necessary for?

It is necessary for: Metabolizing certain nutrients Correcting nutritional deficiencies and Eliminating foods that worsen disease states

Chest tube key points

Keep drainage device below chest If tube disconnects? Then have patient cough and exhale; put it in sterile water, call physician, cover with thin with wicking, so it can breathe

What is a lactovegetarian?

Lactovegetarian (drinks milk but avoids eggs) Vegan (consumes only plant foods)

Lifestyle modifications regarding oxygenation

Lifestyle modifications are usually perceived as difficult for patients because they often have to change an enjoyable habit such as cigarette smoking or eating certain foods. Risk factor modification is important and includes smoking cessation, weight reduction, a low-cholesterol and low-sodium diet, management of hypertension, and moderate exercise. Helping patients acquire healthy behaviors reduces the risk for or slows or halts the progression of cardiopulmonary disease.

What is the only essential fatty acid found in humans?

Linoleic acid is the only essential fatty acid in humans; linoleic and arachidonic acids are manufactured by the body if linoleic acid is available.

Tracheostomy key points

Long term patient that need an artificial airway throat cancer upper airway cancer Special valve is needed, because this lays over the voice box, and the patient can not talk Humidification is also an issue; becomes dry Special oxygen collar to place over the trach Most common complication - upper airway, because secretions collect sterile

Chemical regulation of respiration

Maintains the appropriate rate and depth of respirations based on changes in carbon dioxide, oxygen, and hydrogen ion concentrations in the blood

Chemical regulation

Maintains the rate and depth of respirations based on changes in the blood concentrations of CO2 and O2, and in hydrogen ion concentration (pH)

What strategy can be used to enhance the intake of healthful foods for an adolescent?

Making healthful food choices more convenient and available for the adolescent; this helps prompt the adolescent to make healthier food choices.

Why is it important to enhance nutritional status in cancer patients?

Malignant cells compete with normal cells for nutrients, thus increasing the patient's metabolic needs. It is important to enhance nutritional status to improve the patient's quality of life.

What are enteral feeding precautions and practice?

May be intermittent or continuous Head of bed: minimum 30 degrees, pref. 45 Check placement Flush tubing : Gastric residual? Complications? Dislodged; not patent Are we concerned about blood glucose? Yes, patient can sometimes have high blood sugar

How do medications affect nutrition and metabolism?

Medications - can alter taste and appetite and can interfere with the absorption of certain nutrients

What are diet considerations for adolescents? (12-20 years)

Metabolic demands are high and require more energy Protein, calcium, iron, iodine, folic acid, and vitamin B needs are high 1/4 of dietary intake comes from snacks Increased water consumption is important for active adolescents

What types of metabolic reactions are there?

Metabolic reactions include glycogenolysis, glycogenesis, and gluconeogenesis.

What is metabolism consist of and what is it?

Metabolism consists of anabolic and catabolic reactions Metabolism refers to all of the biochemical reactions within the cells of the body.

What are the methods for confirming placement of a tube?

Methods for confirming placement: X-ray pH Air bolus? - not evidence based practice

What methods are there for oxygenation therapy?

Methods of supply Nasal cannula 1-6 LPM, 24-44% Oxygen mask Simple Mask 6-12 LPM, 35-50% Partial rebreather and non-rebreather 60-90%% at 10-15 L/min Oxymizer 8 LPM, 30-50% Venturi mask - most precise percentage of oxygen 24-50% of oxygen, variable LPM Face tent or trach collar

What are minerals? What is their primary purpose?

Minerals are inorganic elements essential to the body as catalysts in biochemical reactions. We need 100 mg or more daily of macrominerals and 100 mg or less of trace elements. Minerals are catalysts for enzymatic reactions

What is a monounsaturated fatty acid composed of?

Monounsaturated fatty acids have one double bond, whereas polyunsaturated fatty acids have two or more double carbon bonds.

Magnesium

Most of the body's magnesium is found in the bones. Magnesium in smaller amounts is found within the body cells. A very small amount is found in ECF. **EXPECTED LEVELS **1..3-2.1 mEq/L**

Describe pulmonary circulation

Moves blood to and from the alveolar capillary membranes for gas exchange Pulmonary circulation begins at the pulmonary artery, which receives poorly oxygenated mixed venous blood from the right ventricle. Blood flow through this system depends on the pumping ability of the right ventricle. The flow continues from the pulmonary artery through the pulmonary arterioles to the pulmonary capillaries, where blood comes in contact with the alveolar capillary membrane, and the exchange of respiratory gases occurs. The oxygen-rich blood then circulates through the pulmonary venules and pulmonary veins, returning to the left atrium.

Legal guidelines for documentation

Must include date and time Do not scratch out or use white-out on errors Sign with name and title Do not include opinions, give facts, and be specific

What is myocardial ischemia?

Myocardial ischemia: Coronary artery flow to the myocardium insufficient to meet myocardial oxygen demands; results in angina, myocardial infarction (MI) and/or acute coronary syndrome (ACS)

What and where do you administer for enteral feedings?

Nasogastric, gastrostomy, or jejunostomy tubes nose, esophagus, stomach, jejunum

How do negative experiences with certain foods or familiarity with foods affect nutrition and metabolism?

Negative experiences - negative experiences with certain foods or familiarity with foods aids in client preferences

When does negative nitrogen occur in our bodies?

Negative nitrogen occurs in infection, burns, fever, starvation, and trauma.

What controls the process of respiration?

Neural and chemical regulators control the process of respiration.

What does a regular therapeutic diet consist of?

No restrictions

What is used to prevent the use of invasive artificial airways in patients with acute respiratory failure, cardiogenic pulmonary edema, or exacerbation of COPD?

Noninvasive positive-pressure ventilation (NPPV) is used to prevent the use of invasive artificial airways (endotracheal [ET] tube or tracheostomy) in patients with acute respiratory failure, cardiogenic pulmonary edema, or exacerbation of COPD.

How long is the QRS complex?

Normal QRS duration is 0.06 to 0.12 second.

What are normal lung values determined by?

Normal lung values are determined by age, gender, and height.

Normal sinus rhythm

Normal sinus rhythm originates at the sinoatrial (SA) node and follows the normal sequence through the conduction system.

What lifestyle factors affect oxygenation

Nutrition Exercise Smoking Substance abuse Stress Environmental factors Higher weight - increased oxygen need Alcohol suppresses the respiratory system Stress - increased basal metabolic rate, increased oxygen, exhaustion Smokers 10x greater risk for lung cancer Women - also higher risk for blood clots

Nutrition regarding oxygenation

Nutrition: Severe obesity decreases lung expansion, and increased body weight increases tissue oxygen demands. Patients who are morbidly obese and/or malnourished are at risk for anemia. Diets high in carbohydrates play a role in increasing the carbon dioxide load for patients with carbon dioxide retention.

How long should you suction?

Only suction for about 10 seconds

Artificial airway: When would you use oral airway?

Oral airway: Prevents obstruction of the trachea by displacement of the tongue into the oropharynx

Suctioning techniques: oropharyngeal and nasopharyngeal

Oropharyngeal and nasopharyngeal: Used when the patient can cough effectively but is not able to clear secretions

Suctioning techniques: orotracheal and nasotracheal

Orotracheal and nasotracheal: Used when the patient is unable to manage secretions

What is a ovolactovegetarian?

Ovolactovegetarian (avoids meat, fish, and poultry, but eats eggs and milk)

Oxygenation precautions

Oxygen is a highly combustible medical gas and should be used in accordance with federal, state, and local regulations. It can easily cause a fire in a patient's room if it contacts a spark from an open flame or electrical equipment. With increasing use of home oxygen therapy, patients and health care professionals need to be aware of the dangers of combustion.

What is the formula for oxygen transport?

Oxygen transport = Lungs + cardiovascular (CV) system

What does the P wave represent?

P wave represents contractions thru both atria

Physical examination for oxygenation: Palpation

Palpation: Chest, feet, legs, pulses

Why do patients use incentive spirometry?

Patients use incentive spirometry to prevent or treat atelectasis.

How long each day should a person exercise at minimum? Why?

People who exercise for 30 to 60 minutes daily have a lower pulse rate and blood pressure, decreased cholesterol level, increased blood flow, and greater oxygen extraction by working muscles.

What are examples of peptic ulcer treatments when medical nutrition therapy is involved?

Peptic ulcer treatments: Avoid caffeine. Avoid spicy foods. Avoid aspirin, NSAIDs, they can irritate gastric linings Consume small, frequent meals.

What are peptic ulcers commonly caused by?

Peptic ulcers are most commonly caused by helicobacter pylori

Physical examination for oxygenation: Percussion

Percussion: thoracic expansion compare side to side over the lung fields and want to hear resonance, not dullness diaphragmatic excursion

A nurse is explaining to the parents of a 14 months old toddler what physical and cognitive development they can expect from now until their son is 3 years old. A. Physical development -identify gross or fine motor skills the parents can express at specific ages B. Cognitive development -describe parameters the parents can expect to observe during the toddler stage

Physical development -15 months gross motor skills: --walks without helps, creeps up stairs -15 months fine motor skills: --uses cup well, builds tower of two blocks -18 months gross motor skills: --assumes standing position, jumps in place with both feet 18 month fine motor skills: --manages spoon without rotations, turns pages in book two or three at a time -2 years gross motor skills: --walks up and down steps -2 years fine motor skills: --builds a tower with six or seven blocks -2.5 years gross motor skills: --jumps with both feet, stands on one foot momentarily -2.5 years fine motor skills -- draws circles, has good hand-finger coordination Cognitive Development -during toddler stage: --object permanence --memories of events that relate to them --domestic mimicry (playing house) --symbolization of objects and people --use of 400 words --use of two-to-three word phrases

List physical and cognitive development expectations for young adulthood

Physical development -completion of growth -peak in physical senses -peak in cardiac output, efficiency -optimal muscles function -gradual decline in metabolic rate -high libido (men) -eventual peak in libido (women) -optimal childbearing -pregnancy-related changes Cognitive development -improvement in critical thinking -peak in memory -increased ability for creative thoughts -relevance of values/norms of friends

What should the physical examination consist of when assessing for nutrition?

Physical examination: Skin, mucous membranes, muscle tone, hair, teeth, oral cavity

Factors affecting oxygenation: physiological

Physiological factors: Decreased oxygen-carrying capacity - high altitude Hypovolemia - dehydration Decreased inspired oxygen concentration Increased metabolic rate - metabolism Inhaling something that should not be inhaled, such as a helium balloon anemia

How would a nurse assess the patient for adequate swallowing?

Place fingers on the patient's throat at the level of the larynx and ask the patient to swallow; the nurse should be able to palpate the movement of the pharynx

When is a pneumococcal vaccine recommended?

Pneumococcal vaccines - recommended for patients 60+ and those who have chronic diseases

What is a polysaccharide?

Polysaccharides such as glycogen are made up of many carbohydrate units; they are complex carbohydrates.

Why is positive nitrogen required of our bodies?

Positive nitrogen is required for growth, normal pregnancy, wound healing, and vital organ functioning.

A nurse is caring for a client in a spinal cord injury rehabilitation center following head and neck injuries he sustained while riding his bicycle. The client had surgery during the acute phase of treatment to relieve intracranial pressure and to stabilize his cervical spine. Now, he and his spouse are learning essential self-management strategies. List each of the three levels of prevention with an example of each level from this client's history or from what this client might have done to prevent this injury and its life-altering consequences.

Primary: take various courses, read about bicycle safety (wear a helmet, use reflective accessories and lights for visibility to drivers, follow the rules of the road for cyclists) Secondary: emergency care, surgery Tertiary: rehabilitative care, learning self-management procedures, strategies

What are the three types of standards created by HIPPA?

Privacy Security Administrative simplification only members of the healthcare team involved in care clients have the right to access their own records staff many not photocopy unless an authorized exchange or transfer of info is occurring

How can promotion of lung expansion be achieved?

Promotion of lung expansion can be achieved by mobility, positioning, incentive spirometry, and chest tube insertion.

Breathing exercises for restorative care

Pursed-lip breathing Diaphragmatic breathing

For prescriptions or orders...

REPEAT BACK AND ASK "IS THAT CORRECT?"

Cerebral Cortex

Regulates the voluntary control of respiration

Why is regulation of respiration necessary?

Regulation of respiration is necessary to ensure sufficient oxygen intake and carbon dioxide elimination to meet the demands of the body (e.g., during exercise, infection, or pregnancy).

A nurse educator is teaching a module on biotransformation as a phase of pharmokinetics during nursing orientation to a group of newly licensed nurses. A. Related content - list areas of the body where biotransformation takes place B. Underlying principles - list factors that influence the rate of biotransformation

Related content -biotransformation (metabolism) changes medications into less active forms or inactive forms by the action of enzymes. This occurs primarily in the liver, but also takes place in the kidneys, lungs, bowel, and blood Underlying principles -age --infants have limited medication-metabolizing capacity. The aging process also can influence medication metabolism, but varies from individual to individual. Hepatic medication metabolism tends to decline with age -increase in certain medication-metabolizing enzymes - This can cause a particular medication to be metabolized sooner, requiring an increase in dosage of that medication to maintain a therapeutic level. It can also cause an increase in the metabolism of other medications that are being used concurrently -first-pass effect - some medications are inactivated on their first pass through the liver and must be given by a non-enteral route because of their high first-pass effect. These medications usually are given by routes such as sublingual or IV -similar metabolic pathways- when two medications are metabolized by the same pathway, they can interfere with the metabolism of one or both of the medications. In this way, the rate of metabolism can be decreased for one or both of the medications leading to medication accumulation -nutritional status - a malnourished client may be deficient in the factors that are necessary to produce specific medication-metabolizing enzymes. Consequently, medication metabolism may be impaired

When the diaphragm relaxes and the internal intercostal muscles contract, what happens?

Relaxation of the diaphragm and contraction of the internal intercostal muscles allow air to escape from the lungs.

How does religious and cultural practices affect nutrition and metabolism?

Religious and cultural - may guide food preparation and choices

What is afterload?

Resistance to left ventricular ejection

What is resting energy expenditure? (REE)

Resting energy expenditure (REE) (aka resting metabolic rate)—the amount of energy that an individual needs to consume over a 24-hour period for the body to maintain all of its internal working activities while at rest

The nurse is working on a pulmonary unit at the local hospital. The nurse is alert to one of the early signs of hypoxia in the client which is?

Restlessness

What are diet considerations for school age children? (6-12 years)

School-age children need supervision to consume adequate protein and vitamins C and A School-age children tend to eat foods high in carbohydrates, fats, and salt

What should we assess for when screening?

Screening: Height Weight (rapid weight gain or weight loss?) BMI Primary diagnosis Comorbidities

Hypercalcemia

Serum calcium level greater than 10.5 mg/dL

Hypomagnesemia

Serum magnesium level less than 1.3 mEq/L

For patients with short bowel syndrome, which is a malabsorption syndrome, what would the medical nutrition therapy include?

Short bowel syndrome is when the intestinal surface is decreased Treat lifetime EN or PN

Change-of-shift reports need to include:

Significant info re: health problems New info: recent changes to meds, treatments, etc. Logical sequence No gossip or personal opinions

What is a carbohydrate? What is its main function?

Simple and complex saccharides; main source of energy/fuel

What is a simple carbohydrate?

Simple carbohydrates are monosaccharides and disaccharides.

Supplemental oxygen key points

Skin assessment: -Masks/ nasal cannulas cause skin breakdown Humidity: -Important, have to use if patient has over 4 L of O2 Bag should not be flat or deflated: -1/3 - 2/3 inflated before putting on -Flat? Not getting oxygen, but rebreathed CO2 -When in, 1/3 to half full

What are sources of protein?

Sources include: ground beef, whole milk, and poultry

What are sources of fat?

Sources include: olive oil, baked potatoes, and brown rice

What are sources of carbohydrates?

Sources include: whole grain breads, baked potatoes, and brown rice

What is medical nutrition therapy? (MNT)

Specific nutritional therapy usage for treating illness, injury, or a certain condition

What does spirometry measure?

Spirometry measures the volume of air entering or leaving the lungs.

How do you treat dyspnea?

Start with least invasive Dyspnea? Treat the cause maintain their airway not well hydrated? Secretions will be thick and will not be able to expel them breathing techniques humidification - helps dry mucosa, because O2 is drying; tap water attached to O2 nebulization - albuterol; aerosolized med, breath in (dr. order needed; ordered may be as needed and that's when our judgement comes in) coughing and deep breathing - cough every two hours and deep breath chest percussion / vibration - helps loosen secretions (if not successful, yankhaur suction is needed)

What is stroke volume affected by?

Stroke volume affected by - cardiomyopathies, blood disorders, hypotension, hemorrhage

Is suction a clean procedure or a sterile procedure?

Suction is a clean procedure, not sterile

Suction guidelines and tips

Suction trachea before the oropharynx hyper-oxygenate before suctioning monitor pulse ox hold breath and count in head (helps because the patient can not breath during this procedure) Inhale while inserting 150 mg of mercury

Why is suctioning necessary?

Suctioning is necessary when patients are unable to clear respiratory secretions from the airways by coughing or other less invasive procedures. Suctioning techniques include oropharyngeal and nasopharyngeal suctioning, orotracheal and nasotracheal suctioning, and suctioning of an artificial airway

A nurse is caring for a patient who has sustained a head injury and whose level of consciousness fluctuates. The provider prescribes a full liquid diet progressing to a pureed diet as tolerated. Before initiating feedings, it is essential that this patient undergo what?

Swallowing examination; patients at high risk for aspiration include those with a decreased level of consciousness. This patient has some periods of decreased alertness, thus a swallowing examination is essential to determining his ability to ingest food safely by mouth.

s/s for FVE

Tachycardia, bounding pulse, HTN, tachypnea, confusion, muscle weakness, weight gain, crackles, edema, and distended neck veins

Types of report

Telephone Transfer Incident

Noninvasive: TB skin test Holter monitor ECG Thallium stress test EPS PFT

Thallium - injected into vessels and see movement of heart EPS - electro physical study - catheter inserted in or near heart - see reaction of heart PFT - pulmonary function test

HITECH

The Health Information Technology for Economic and Clinical Health (HITECH) Act, enacted as part of the American Recovery and Reinvestment Act of 2009, was signed into law on February 17, 2009, to promote the adoption and meaningful use of health information technology

What is HIPPA?

The Health Insurance Portability and Accountability Act of 1996 (HIPPA) established new standards for the confidentiality, security, and transmissibility of health care information.

What does the QRS complex represent?

The QRS complex represents the impulse traveling thru the ventriclesT

What does the QT interval represent?

The QT interval represents the time needed for ventricular depolarization and repolarization

Perfusion

The ability of the cardiovascular system to pump oxygenated blood to the tissues and return deoxygenated blood to the lungs

Where is oxygen exchanged for carbon dioxide?

The airways of the lung transfer oxygen from the atmosphere to the alveoli, where the oxygen is exchanged for carbon dioxide.

What is cardiac output? (CO)

The amount of blood ejected from the left ventricle each minute

Why does the body require protein to be incorporated solely through diet?

The body does not synthesize indispensable amino acids; thus they need to be provided in the diet. The body synthesizes dispensable amino acids.

Why does the body require fuel?

The body requires fuel to provide energy for cellular metabolism and repair, organ function, growth, and body movement. Life-sustaining activities include breathing, circulation, heart rate, and temperature.

What does the cardiac system deliver?

The cardiac system delivers oxygen, nutrients, and other substances to the tissues and facilitates the removal of cellular metabolism waste products by way of blood flow through other body systems such as respiratory, digestive, and renal.

Closed drainage system in a single chamber

The closed drainage system in a single chamber. The chamber serves as both collector and water seal.

Conduction system

The conduction system originates with the SA node, the "pacemaker" of the heart. The SA node is in the right atrium next to the entrance of the superior vena cava. Impulses are initiated at the SA node at an intrinsic rate of 75 cardiac action potentials per minute in an adult at rest. Electrical impulses are transmitted through the atria along intra-atrial pathways to the atrioventricular (AV) node. The AV node mediates impulses between the atria and the ventricles. It assists atrial emptying by delaying the impulse before transmitting it through the bundle of His and the ventricular Purkinje network.

When the diaphragm and external intercostal muscles contract, what happens?

The diaphragm and external intercostal muscles contract to create a negative pleural pressure and increase the size of the thorax for inspiration.

Dry chest tube system

The dry chest tube system, which does not use water in the suction chamber. The automatic control valve (ACV) continuously balances the force of the suction with the atmosphere.

Where does the exchange of gases occur?

The exchange of gases occurs between environment and blood.

What is the goal of nutrition in cancer patients?

The goal of nutrition treatment in cancer patients is to meet the increased metabolic demands. Enhanced nutritional status often improves a patient's quality of life.

Where is the primary area of absorption?

The intestine is the primary area of absorption

The left ventricle pumps blood through which circulation?

The left ventricle pumps blood through the systemic circulation.

What does the left ventricle pump?

The left ventricle pumps oxygenated blood through the systemic circulation

How long is the PR interval?

The length of the PR interval is 0.12 to 0.20 second.

Explain the process how oxygen is exchanged for carbon dioxide

The lungs transfer oxygen from the atmosphere to the alveoli, where the oxygen is exchanged for carbon dioxide.

What is the major inspiratory muscle of respiration?

The major inspiratory muscle of respiration is the diaphragm

What is the most common complication of a tracheostomy tube?

The most common complication of a tracheostomy tube is partial or total airway obstruction caused by buildup of respiratory secretions.

What is the most effective position for promotion of lung expansion? Why?

The most effective position for promotion of lung expansion is the 45-degree semi-Fowler's position. This position uses gravity to assist in lung expansion and reduces pressure from the abdomen on the diaphragm.

What does the oxygen transport system consist of?

The oxygen transport system consists of the lungs and cardiovascular system.

A nurse is performing a nutritional assessment. When obtaining and interpreting anthropometric values, the nurse should recognize which of the following?

The patient should be weighed on the same scale at the same time everyday; weighing a patient on the same scale each day at the same times provides the most consistent data for gauging trends in the patient's weight, as shifts in fluid intake and output can alter weight significantly. The patient should also be weighed with the same amount of clothing/ and or linen each time

Do artificial airways present a risk?

The presence of an artificial airway places a patient at high risk for infection and airway injury. Use clean technique for oral airways, but use sterile technique in caring for and maintaining endotracheal and tracheal airways to prevent health care-associated infections (HAIs).

What is the primary function of the lungs?

The primary function of the lungs is to transfer oxygen from the atmosphere into the alveoli and carbon dioxide out of the body as a waste product.

What is the primary function of the heart?

The primary functions of the heart are to deliver deoxygenated blood to the lungs for oxygenation, and oxygen and nutrients to the tissues.

Ventilation

The process of moving gases into and out of the lungs

What four components are necessary for ventilation, perfusion, and exchange of respiratory gases?

The respiratory muscles, pleural space, lungs, and alveoli are essential for ventilation, perfusion, and exchange of respiratory gases.

The right ventricle pumps blood through which circulation?

The right ventricle pumps blood through the pulmonary circulation.

What does the right ventricle pump?

The right ventricle pumps deoxygenated blood through the pulmonary circulation

What are diet considerations for young adults? (20-35 years) and middle adults? (35-65 years)

There is a decreased need for most nutrients (except during pregnancy) calcium and iron are essential minerals for women good oral health is important

What do the parasympathetic fibers decrease and where do they originate from?

They decrease the rate of impulse generation and the speed of transmission and they originate from the vagus nerve

What do the sympathetic fibers increase the rate of?

They increase the rate of impulse generation and the speed of transmission

How does noninvasive positive-pressure ventilation (NPPV) work to prevent or treat atelectasis?

This prevents or treats atelectasis by inflating the alveoli, reducing pulmonary edema by forcing fluid out of the lungs back into circulation, and improving oxygenation in those with sleep apnea.

What are the three steps involved in the process of oxygenation?

Three steps are involved in the process of oxygenation: ventilation, perfusion, and diffusion.

What three things influence the capacity of the blood to carry oxygen?

Three things influence the capacity of the blood to carry oxygen: the amount of dissolved oxygen in the plasma, the amount of hemoglobin, and the tendency of hemoglobin to bind with oxygen.

What does the oxygen travel through in order to transfer to the blood, and then carbon dioxide to transfer to the alveoli?

Through the alveolar capillary membrane, oxygen transfers to the blood, and carbon dioxide transfers from the blood to the alveoli.

What is the primary purpose for asking a patient to keep a 3-7 day food diary?

To assess the pattern of intake and compare with daily reference intakes

What is the goal of oxygenation therapy?

To prevent or relieve hypoxia by delivering oxygen at concentrations greater than ambient air (21%).

Suctioning techniques: tracheal

Tracheal Used with an artificial airway

Artificial airway: When would you use tracheostomy airways?

Tracheostomy: Long-term assistance, surgical incision made into trachea

Where do triglycerides circulate in the body and what are they composed of?

Triglycerides circulate in the blood and are composed of three fatty acids attached to a glycerol.

True of false: Fats are the most calorie-dense nutrients.

True

True of false: Patients unable to digest or absorb enteral nutrition or are in highly stressed physiological states:

True

True of false: Patients with pulmonary diseases have decreased surfactant production causing atelectasis

True

True of false: Rhythmic relaxation and contraction of the atria and ventricles depend on continuous, organized transmission of electrical impulses. The cardiac conduction system generates and transmits these impulses.

True

True or false: Carbohydrate ingestion is best if supplied by complex carbs

True

True or false: Conditions and diseases that change the structure and function of the pulmonary system alter respiration.

True

True or false: Early ambulation studies indicate that the therapeutic benefits of activity include an increase in general strength and lung expansion.

True

True or false: Fatty acids may be essential or nonessential.

True

True or false: In general, when energy requirements are completely met by caloric intake in food, weight does not change.

True

True or false: Oxygen is a medication that requires a Dr.'s order

True

True or false: Sympathetic and parasympathetic nerve fibers innervate all parts of the atria and ventricles and the SA and AV nodes.

True

True or false: The longer the material stays in the large intestines, the firmer are the feces.

True

True or false: The thickness of the alveolar capillary membrane affects the rate of diffusion.

True

True or false: When the kilocalories ingested exceed a person's energy demands, the individual gains weight. If the kilocalories ingested fail to meet a person's energy requirements, the individual loses weight

True

True or false: in CPR - circulation is always first.

True

Myocardial pump

Two atria and two ventricles As the myocardium stretches, the strength of the subsequent contraction increases (Starling's law).

Two chamber system

Two-chamber system, which permits liquid to flow into the collection chamber as air flows into the water-sealed chamber.

A nurse is introducing a group of nursing students to the various approaches to problem-oriented documentation. List the three common methods of problem-oriented charting with definitions of their acronyms

Underlying principles SOAP S - subjective data O - Objective data A - Assessment (includes a nursing diagnosis based on the assessment P - Plan PIE P - Problem I - Intervention E - Evaluation DAR (focus charting) D - Data A - Action R - Response

Myocardial blood flow

Unidirectional through four valves S1: mitral and tricuspid close S2: aortic and pulmonic close

Vaccinations?

Vaccinations: Influenza pneumococcal

What could variations in tidal volume and other lung volumes be associated with?

Variations in tidal volume and other lung volumes are associated with alterations in patients' health status or activity, such as pregnancy, exercise, obesity, or obstructive and restrictive conditions of the lungs.

What does a vegetarian diet consist of?

Vegetarian diet consists predominantly of plant foods

How is ventilatory support achieved?

Ventilatory support is achieved using a variety of modes, including continuous positive airway pressure (CPAP) and bilevel positive airway pressure (BiPAP).

What are vitamins? What is their primary purpose?

Vitamins are organic substances present in small amounts in foods that are essential to normal metabolism. Fat-soluble vitamins: A, D, E, K. Water-soluble vitamins: C and B complex. Vitamins are essential for metabolism.

When is medical nutrition therapy most effective?

When it is combined with a collaborative health care team and dietitian

When would you administer parenteral nutrition to a patient?

When the patient is unable to use their gastrointestinal tract to acquire nutrients

Can parenteral feedings be given by a peripheral line or a central line?

Yes

Do I need to maintain a sterile environment in a trach?

Yes, trachs - keep inside sterile, because it is directly in the line of breathing

What is a protein? What is its main function?

amino acids; necessary for nitrogen balance and are essential for tissue growth, maintenance, and repair. Proteins provide nitrogen. Nitrogen balance is essential for a normal functioning body.

How often should you get your cholesterol evaluated?

at age 20, then at least q5 years

How often should you get your blood glucose evaluated?

at age 45, then at least q3 years

What age should you first get a colorectal exam? How often after that?

at age 50, then annually

How often should you get your blood pressure evaluated?

at least every 2 years, annual if previously elevated

What does a diabetic therapeutic diet consist of?

balanced intake of protein, fats, and carbohydrates of about 1,800 calories; include a balanced intake of carbohydrates, fats, proteins varied caloric recommendations to accomodate patient's metabolic demands

A nurse is reviewing with a group of nursing students how to perform postural drainage. List the specific positions that facilitate secretion drainage from at least eight specific lung areas

both lobes in general: high Fowler's apical segments of both lobes: sitting on the side of the bed right upper lobe, anterior segment: supine with head elevation right upper lobe, posterior segment: on the left side with a pillow under the right side of the chest right middle lobe, anterior segment: 3/4 supine with dependent lung in Trendelenburg Right middle lobe, posterior segment: prone with thorax and abdomen elevation Right lower lobe, lateral segment: on the left side in Trendelenburg Left upper lobe, anterior segment: supine with head elevation Left upper lobe, posterior segment: on the right side with a pillow under the left side of the chest left lower lobe, lateral segment: on the right side in Trendelenburg Both lower lobes, anterior segments: supine in Trendelenburg Both lower lobes, posterior segments: prone in Trendelenburg

Neural regulation

cerebral cortex regulates the voluntary control of respiration by delivering impulses to the respiratory motor neurons by way of the spinal cord

Health history for oxygenation patients - what do we include?

chest pain dyspnea wheezing respiratory infection health risks fatigue cough smoking allergies medications environment/geographical exposures

What lab values should be taken when assessing nutrition?

cholesterol, triglycerides, hemoglobin, electrolytes, albumin, nitrogen levels,

What does a mechanical soft therapeutic diet consist of?

clear and full liquids plus diced or ground foods, with addition of all cream soups, ground or finely diced meats, flaked fish, cottage cheese, cheese, rice, potatoes, pancakes, light breads, cooked vegetables, cooked or canned fruits, bananas, soups, peanut butter, eggs (not fried)

What does a pureed therapeutic diet consist of?

clear and full liquids plus pureed meats, fruits, vegetables, and scrambled eggs, and mashed potatoes and gravy

What are some appropriate food choices for a patient put on a full liquid diet?

clear liquids plus liquid dairy products, all juice, pureed vegetables, ice-cream, strained or blended cream soups, custards, refines cooked cereals, vegetable juice, sherbets, puddings, frozen yogurt

What does a full liquid therapeutic diet consist of?

clear liquids plus liquid dairy products, all juice, pureed vegetables, ice-cream, strained or blended cream soups, custards, refines cooked cereals, vegetable juice, sherbets, puddings, frozen yogurt

Risk Factors for Hypocalcemia

diarrhea, malabsorption, thyroidectomy

Tx for Hypermagnesemia

diuresis (loop diruretic)

When teaching the parents of a toddler about feeding and eating, what should the nurse include regarding safety measures?

do not offer the child raw vegetables; raw vegetables, as well as hot dogs, grapes, nuts, popcorn, and candy have been implicated in choking deaths and should be avoided at least until the child is 3 years old

What does a gluten free therapeutic diet consist of?

eliminating wheat, oats, rye, barley, and their derivatives

How often should you have a dental assessment?

every 6 months

What does a soft/low residue therapeutic diet consist of?

foods that are low in fiber and easy to digest, such as pastas, casseroles, moist tender meats, and canned cooked fruits and vegetables, deserts, cakes, and cookies without nuts or coconut

Risk factors/Causes for FVE

heart failure, cirrhosis, renal disease, excessive sodium intake

What are diet considerations for infants? (Birth - 1 year)

high energy requirements breast milk (preferred) or formula to provide 108 kcal/kg of weight for the first 6 months 98 kcal/kg of weight the second 6 months solid food starting after 6 months of age no cow's milk or honey for the first year

Causes and Risk Factors for Hypercalcemic pts

hyperparathyroidism, bone cancer, and Paget's disease

s/s for FVD

hypothermia, tachycardia, thready pulse, hypotension, ortho hypo, tachypnea, and hypoxia Dizziness, confusion, weakness and fatigue oliguria, diminished cap refill, cool clammy skin, and decreased skin turgor

What does the PR interval represent?

impulses traveling from the SA node thru the AV node, Bundle of His, and Perkinje fibers.

Risk Factors for Hypermagnesemia

kidney failure, laxatives or antacids containing magnesium, and adrenal insufficiency

Nursing care for Hypercalcemic pts

limit calcium, encourage fluids and fiber

What does parenteral nutrition include?

lipids, electrolytes, minerals, vitamins, dextrose, and amino acids

What does a clear liquid therapeutic diet consist of?

liquids that leave little residue (clear fruit juices, gelatin, broth, coffee, tea, carbonated beverages, fruit ices, popsicles)

What do you assess for when taking intake and outputs?

monitor I&O for clients who have fluid and electrolyte imbalances weight clients each day at the same time, after voiding, and while wearing the same type of clothes if using bed scales us the same amount of linen each day, and reset the scaled to zero if possible

S/S for Hypermagnesemia

muscle paralysis, diminished DTRs, bradycardia, hypotension

What should you assess for subjectively and objectively to indicate poor nutrition?

nausea, vomiting, diarrhea, constipation flaccid muscles mental status changes loss of appetite change in bowel pattern spleen, liver enlargement dry, brittle hair loss of subcutaneous fat dry, scaly skin inflammation, bleeding gums poor dental health dry, dull eyes enlarged thyroid prominent protrusions over bony areas weakness changes in weight poor posture

What does a low-sodium therapeutic diet consist of?

no added salt or 1-2 g of sodium

What does a low cholesterol therapeutic diet consist of?

no more than 300 mg/day of dietary cholesterol

Cascade cough

on exhale have patient forcefully cough to try to pull up secretions from below

What tests should females get regularly?

pap smear for cervical cancer breast cancer screening

How do you assist preventing aspiration in patients?

position in fowlers position or in a chair support the upper back, neck, and head have clients tuck their chins when swallowing to help propel food down the esophagus observe for aspiration and pocketing of food in the cheeks or other areas of the mouth observe for signs of dysphagia, such as coughing, choking, gagging, and drooling of food keep clients in semi-fowlers position for at least 1 hr after meals provide oral hygiene after meals and snacks

With each gram of a protein, how many kcal's does it produce?

providing 9 kcal/g.

What does a dysphagia therapeutic diet consist of?

pureed food and thickened liquids

How often should a male get a routine physical exam?

q5 years for men until age 40, then more frequently, BMI at each visit

What is a fat? What is its main function?

saturated, polyunsaturated, and monunsaturated; calorie dense

Huff cough

say the word huff on exhale in order to loosen secretions in upper airway

Hypermagnesemia

serum magnesium level greater than 2.1 mEq/L

S/S for Hypercalcemic pts

shortened QT and ST segments, weakness, decreases reflexes

S/S for Hypomagnesemia

similar to hypocalcemia: +Chvoteks(facial nerve) +Trousseau's +Muscle spasms at rest increased nerve impulse transmission (hyperactive deep tendon reflexes, parasthesias, muscle tetany) Dysrhythmias, tachycardia, HTN

What tests should males get regularly?

testicular exams prostate exams

Tidal volume

the amount of air exhaled after normal inspiration

Residual volume

the amount of air left in the alveoli after a full expiration

Forced vital capacity

the maximum amount of air that can be removed from the lungs during forced expiration

What does the central nervous system control regarding respiration?

the respiratory rate, depth, and rhythm

What does the cerebral cortex regulate regarding respiration?

the voluntary control of respiration The cerebral cortex regulates the voluntary control of respiration by delivering impulses to the respiratory motor neurons by way of the spinal cord.

What are diet considerations for toddlers? (12 months to 3 years) and preschoolers? (3-6 years)

toddlers and preschoolers need fewer calories per kg of weight than infants toddlers and preschoolers need increased protein from sources other than milk calcium and phosphorus are important for bone health

Hypocalcemia

total serum calcium level less than 9mg/dL

Risk Factors/ Causes for FVD

vomiting, NG suctioning, diarrhea, bleeding, diaphoresis, decreased intake

As feces move toward the rectum, what nutrient is absorbed in the mucosa?

water

What does a high-fiber therapeutic diet consist of?

whole grains, raw and dried fruits, addition of fresh uncooked fruits, steamed vegetables, bran, oatmeal, and dried fruits


Conjuntos de estudio relacionados

Chapter 38: Caring for Clients With Cerebrovascular Disorders

View Set

Applications with Standard Normal Distribution Assignment and Quiz 80%

View Set

Chapter 30 - China: The World's Most Populous Country

View Set

DECA Finance Cluster Exam #1 Corrections

View Set

Electromagnetic spectrum and light

View Set

Chapter 41: The Child with an Integumentary Disorder/Communicable Disease - ML5

View Set